PT 365 Questions

Pataasin ang iyong marka sa homework at exams ngayon gamit ang Quizwiz!

A physical therapist inspects a burn obtained as a result of iontophoresis. The therapist describes the burn as an alkaline reaction. Which of the following factors would MOST likely be the cause of this type of adverse reaction? 1. Sodium hydroxide forming under the cathode 2. Hydrochloric acid-forming under the anode 3. The size of the cathode being larger than the anode 4. An increase in the amount of space between the electrodes

5. Answer: 1 • Rationale: 1. A patient may have an alkaline reaction from the iontophoresis treatment as a result of sodium hydroxide forming under the negative electrode (cathode). 2. A patient may have an acidic reaction from the iontophoresis treatment as a result of hydrochloric acid forming under the positive electrode (anode). 3. The electrode containing the ion solution is referred to as the active electrode. A second electrode, referred to as the dispersive electrode, is placed away from the active electrode. The likelihood of a burn can be reduced by increasing the size of the cathode relative to the anode. 4. The recommended spacing between the active and dispersive electrodes should be minimally equivalent to the diameter of the active electrode. As the spacing between the electrodes increases, the current density in the superficial tissues decreases, resulting in a diminished risk for burns.

A physical therapist analyzes the gait of a child with spastic diplegia cerebral palsy in the school setting. Based on this diagnosis, which muscle group is MOST likely to be shortened? 1. Hip adductors 2. Knee extensors 3. Hip abductors 4. Ankle dorsiflexors

5. Answer: 1 • Rationale: 1. During gait the combination of excessive hip flexion, adduction, and internal rotation produces a scissoring pattern. This type of gait deviation results primarily due to the presence of spasticity. Common management techniques of spasticity in this population include muscle lengthening or release of the musculature. Skeletal muscle relaxants are also used to control spasticity and can be given orally, intrathecally, or through injection at the motor point. 2. The knee flexors, not the knee extensors, are shortened as a result of the spasticity seen in children with spastic diplegia cerebral palsy. 3. The hip adductors, not the hip abductors, are shortened in children with spastic diplegia cerebral palsy. 4. The plantar flexors, not the dorsiflexors, are shortened in children with spastic diplegia cerebral palsy resulting in limited active ankle dorsiflexion. Ankle-foot orthoses are commonly used to position the foot in dorsiflexion to facilitate foot clearance during the swing phase of gait or to decrease genu recurvatum.

A physical therapist directs a physical therapist assistant to measure the seat depth for a patient as part of the assessment for appropriate fit of a wheelchair. How many inches should the therapist subtract from the standard measurement taken for wheelchair depth? 1. 1 inches 2. 2 inches 3. 4 inches 4. 6 inches

5. Answer: 2 • Rational: 1. One inch of space may result in the patient experiencing increased pressure in the popliteal area or even potentially compromised circulation since the amount of space between the front edge of the seat and the popliteal space is less than the recommended amount of two inches 2. Two inches of space between the front edge of the seat and the popliteal space is the recommended amount of space. This distance corresponds to the width of three or four fingers. 3. Four inches of space may result in the patient experiencing decreased trunk stability, increased weight bearing on the ischial tuberosities due to the body weight being shifted posteriorly secondary to the lack of support to the thighs, and poor balance since the base of support has been reduced 4. Six inches of space would serve to exacerbate the difficulties discussed in option 3

A physical therapist measures a patient's shoulder complex medial rotation in the standard position. The therapist records the patient's shoulder medial rotation as 0-70 degrees and classifies the end-feel as firm. Which portion of the joint capsule is PRIMARILY responsible for the firm end-feel? 1. 1. Anterior 2. 2. Posterior 3. 3. Inferior 4. 4. Superior

5. Answer: 2 • Rationale: 1. A firm end-feel caused by the anterior joint capsule would most often be associated with lateral rotation of the glenohumeral joint as the humeral head slides anteriorly on the glenoid fossa. 2. A firm end-feel caused by the posterior joint capsule would most often be associated with medial rotation of the glenohumeral joint as the humeral head slides posteriorly on the glenoid fossa. 3. A firm end-feel caused by the inferior joint capsule would most often be associated with flexion and abduction of the glenohumeral joint. In flexion, the humeral head moves posteriorly and inferiorly, and in abduction, the humeral head moves inferiorly. 4. A firm end-feel caused by the superior joint capsule would most often be associated with extension and adduction of the glenohumeral joint. In extension, the humeral head moves anteriorly and superiorly, and in adduction, the humeral head moves superiorly.

A physical therapist evaluates a patient who is six months pregnant and reports right knee pain. The therapist would like to refer the patient for further diagnostic imaging before starting treatment, but is concerned for the risk of radiation exposure to the developing fetus. Which of the following imaging techniques would have the GREATEST risk of radiation exposure? 1. 1. X-ray 2. 2. Fluoroscopy 3. 3. Magnetic resonance imaging 4. 4. Ultrasound

5. Answer: 2 • Rationale: 1. X-ray is a radiographic image commonly used to assist with the diagnosis of issues related to the bones, such as fractures, dislocations, arthritis, and bone infections. An x-ray uses radiation to penetrate the body and create a two-dimensional picture. However, the dose of radiation used is low and is generally a safe procedure for a pregnant patient, especially since the x-ray beams are directed at the knee and not the abdomen. 2. Fluoroscopy is designed to show motion within the body with the use of x-ray imaging and injection of a contrast dye. The technique permits objects placed between a fluorescent screen and a roentgen tube to become visible. Instead of a single ×-ray image being taken, the x-ray beam is passed through the body continuously to allow for the visualization of movement. Because the x-ray beam is continuous, the level of radiation exposure is much higher than the other presented options 3. Magnetic resonance imaging (MRI) is a noninvasive procedure that utilizes magnetic fields and radio waves to produce cross-sectional images of the body. MRI can be used to visualize almost any structure within the body, but is most often used for imaging soft tissue structures such as muscles, menisci, ligaments, tumors, and internal organs. MRI does not use radiation and would therefore be a safe procedure for a pregnant patient. 4. Ultrasound is a noninvasive procedure that uses sound waves to produce images of structures within the body, especially the internal organs (e.g., liver, kidneys). A transducer is placed on the skin and sends sound waves into the body, where they reflect off the internal structures and are then received and processed by the transducer. Ultrasound shows not only an image of a structure but also the movement of that structure since it is performed in real-time. Ultrasound does not use radiation and would therefore be a safe procedure for a pregnant patient.

A physical therapist observes the gait of a child with spastic cerebral palsy. The therapist notes that the child exhibits excessive lordosis during ambulation. Which of the following surgical procedures would BEST address this postural deformity? 1. Hamstrings tendon lengthening 2. Adductor longs tendon lengthening 3. lliopsoas tendon lengthening 4. Lumbar laminectomy

5. Answer: 3 • Rationale: 1. A hamstrings tendon lengthening procedure will restore the length of the hamstrings muscle. This procedure is used to reduce a knee flexion contracture and results in improved knee extension during gait. A hamstrings contracture is likely to result in a posterior pelvic tilt and reduced lumbar lordosis. 2. Lengthening of the adductor longus tendon is often performed with lengthening of the gracilis and iliopsoas in order to reduce hip subluxation in children with cerebral palsy. Lengthening of the adductor muscle group can result in reduction of a scissoring gait pattern. 3. Contracture of the iliopsoas causes an excessive anterior pelvic tilt, which results in increased lordosis of the lumbar spine. Lengthening of the illopsoas reduces the anterior pull on the pelvis and corrects the lordosis. 4. A laminectomy is a surgical procedure that removes the lamina of a vertebra to relieve pressure on the spinal cord or nerves. The procedure is often performed in patients who have spinal stenosis. A laminectomy would not specifically address the muscular pull on the pelvis that is causing the excessive lordosis.

A physical therapist works with a patient recovering from a surgical resection of a portion of the gastrointestinal tract. Which form of medication administration would likely be the MOST problematic for the patient based on the surgical procedure? 1. Injection 2. Intravenous 3. Oral 4. Transdermal

5. Answer: 3 • Rationale: 1. Injection is a parenteral form of medication administration where the drug can be introduced systemically or locally. Specific types of injection include intravenous, intra-arterial, subcutaneous, intramuscular, and intrathecal. 2. Intravenous is a parenteral form of medication administration where the drug is injected into a peripheral vein. This method allows a known quantity of medication to be rapidly introduced into the bloodstream which allows the drug to have an immediate effect. 3. Oral is an enteral form of medication administration where the drug enters the mouth and moves through the gastrointestinal system typically being absorbed in the small intestine. This type of administration is often avoided with any alteration in gastrointestinal function such as a surgical resection of a portion of the gastrointestinal tract. 4. Transdermal is a parenteral form of medication administration where the drug is applied directly to the surface of the skin to allow for absorption through the dermal layers and into the subcutaneous tissues or the peripheral circulation. This method allows for a slow, controlled release of the drug into the body.

A physical therapist attempts to determine the appropriate back height of a wheelchair for a patient that is able to propel the wheelchair independently but has poor trunk control. The therapist records the distance from the seat of the chair to the floor of the axilla as 20 inches. What is the MOST appropriate back height for the patient given the presence of poor trunk control? 1. 17 inches 2. 16 inches 3. 15 inches 4. 14 inches

a. Answer: 1 b. A standard wheelchair has a back height from 16.0-16.5 inches. Back height is determined by measuring from the seat of the chair to the floor of the axilla with the user's shoulder flexed to 90 degrees and then subtracting four inches. A patient's physical and mental abilities and limitations must be considered when making final recommendations for wheelchair prescription. c. Rationale: 1) A back height of 17 inches would be slightly greater than the standard measurement. The extra back height offers the patient the additional support necessary due to the poor trunk control without compromising their ability to propel the wheelchair. 2) A back height of 16 inches would be the standard back height based on the obtained measurement of 20 inches for many patients (i.e., 20 inches - 4 inches). This level of support would not likely be adequate for a patient with poor trunk control. 3) A back height of 15 inches is less than the back height most often used in a standard wheelchair (i.e. 16.0-16.5 inches). This back height would be unacceptable for a patient with poor trunk control. 4) A back height of 14 inches would fall several inches below the inferior angle of the scapula and would not offer nearly the level of support necessary for a patient with poor trunk control.

A patient reports numbness and tingling in the palmar aspect of the thumb, index, and middle fingers. Examination findings reveal pain with resisted pronation and a negative Tinel's sign at the wrist. Which of the following conditions should the therapist MOST likely suspect? 1. Carpal tunnel syndrome 2. Pronator syndrome 3. Anterior interosseous syndrome 4. Cubital tunnel syndrome

a. Answer: 2 b. Median nerve entrapment at the elbow is far less common than at the wrist and is often misdiagnosed as carpal tunnel syndrome. The three sites in the antecubital area where the median nerve can become entrapped include: 1) under the bicipital aponeurosis, 2) between the two heads of the pronator teres muscle or 3) under the flexor digitorum superficialis. Compression of the median nerve in the antecubital region is typically referred to as pronator syndrome. c. Rationale: 1) Carpal tunnel syndrome (CTS) occurs when the median nerve is injured by compression within the carpal tunnel at the wrist. Although the patient in this scenario is reporting paresthesia's in the distribution of the median nerve, Tinel's sign at the wrist was negative. Additionally, pain with resisted pronation would not be anticipated with CTS. Provocation testing findings for CTS typically include a positive Tinel's sign at the wrist and a positive Phalen's test. 2) Pronator syndrome is an entrapment of the median nerve at the elbow. It is often associated with activities that require repetitive pronation while grasping an object. Other potential risk factors include underlying conditions such as hypothyroidism and diabetes. The paresthesia's in the distribution of the median nerve would be expected with both pronator syndrome and carpal tunnel syndrome, however, the negative Tinel's sign at the wrist and pain with resisted pronation are more typical of pronator syndrome. 3) Anterior interosseous syndrome is a neuropathy resulting in an isolated palsy of the muscles innervated by this motor branch of the median nerve. The three muscles innervated by the anterior interosseous nerve are in the deep layer of the anterior forearm and include flexor pollicis longus, index and long fingers of the flexor digitorum profundus, and pronator quadratus. The classic sign of anterior interosseous syndrome is an inability to perform the "OK" sign by approximating the tip of the index finger to the index of the thumb 4) Cubital tunnel syndrome is an entrapment of the ulnar nerve at the elbow where the nerve passes between the medial epicondyle of the humerus and the olecranon process of the ulna. Typical findings would include pain or paresthesia's in the distribution of the ulnar nerve (i.e., ring and little fingers) and a positive Tinel's sign at the elbow.

A physical therapist treats a patient that is seven months pregnant. The therapist positions the patient in a hooklying position and asks the patient to raise their head from the plinth. The therapist notes a bulge in the central abdominal area. Which medical condition is MOST consistent with the described scenario? 1. De Quervain's disease 2. Diastasis recti 3. Thoracic outlet syndrome 4. Piriformis syndrome

a. Answer: 2 b. Pregnant women are encouraged to continue with exercise activity at a moderate rate during a low risk pregnancy. Guidelines permit women to remain at 50-60 percent of their maximal heart rate for approximately thirty minutes per session. Loose clothing is advised to allow for adequate heat loss and adequate fluids are required during exercise. c. Rationale: 1) De Quervain's disease (tenosynovitis) is the result of an inflammatory process involving the tendons and synovium of the abductor pollicis longs and extensor pollicis brevis at the base of the thumb. Onset is typically due to repetitive activities involving thumb abduction and extension and the associated inflammation results in pain located at the base of the thumb within the anatomical snuffbox. De Quervain's tenosynovitis is more prevalent among women, with higher risk among new mothers due to the repetitive lifting and carrying of the infant. 2) Diastasis recti is a separation of the rectus abdominis muscle along the linea alba that can occur during pregnancy. Testing for diastasis recti should be performed on all pregnant women prior to prescribing exercises that require the use of the abdominals. Biomechanical and hormonal changes may cause the separation. Diastasis recti is diagnosed if there is separation greater than the width of two fingers when the woman lifts her head and shoulders off the plinth. Focused abdominal activities and rotational exercises should be avoided until healing has reduced the separation. 3) Thoracic outlet syndrome is a term used to describe a group of disorders that presents with symptoms secondary to neurovascular compression of fibers of the brachial plexus. A patient with thoracic outlet syndrome will present with symptoms based on nerve and/or vascular compression. This condition can occur during pregnancy due to head and neck postural changes. Typical symptoms include diffuse pain in the arm most often at night, paresthesia's in the fingers and through the upper extremities, weakness and muscle wasting, poor posture, edema, and discoloration. 4) Piriformis syndrome refers to a persistent, severe radiating low back and buttock pain spanning from the sacrum to the hip and posterior thigh. During pregnancy, the piriformis may shorten or spasm due to postural changes and hip lateral rotation with gait (waddling gait). The primary symptom is sciatic paresthesia due to nerve entrapment as the sciatic nerve passes under or through the piriformis muscle. Treatment may include correcting alignment such as muscle energy techniques, joint mobilization, self-correction techniques, heat application, and deep tissue massage.

A physical therapist determines through examination that a patient exhibits Poor (2/5) strength of the adductor pollicis and a positive Froment's sign. The therapist should MOST expect a palsy involving which of the following nerves? 1. Median 2. Anterior interosseous 3. Ulnar 4. Posterior interosseous

a. Answer: 3 b. When assessing for Froment's sign, the patient is asked to hold a piece of paper between their thumb and index finger while the therapist attempts to pull the paper awav from the patient. A positive Froment's sign is indicated by the patient flexing the distal phalanx of the thumb due to weakness or paralysis of the adductor pollicis muscle. The adductor pollicis is innervated by the deep motor branch of the ulnar nerve c. Rationale: 1) Median nerve lesions may present with weak pronation of the forearm, weak wrist flexion, paresthesia's in the thumb index, middle, and lateral half of the ring finger, and atrophy of the thenar muscles, depending on the site of entrapment. Compression of the median nerve in the carpal tunnel is the most common site of entrapment. 2) The anterior interosseous nerve is a motor branch of the median nerve that innervates the muscles in the deep layer of the anterior forearm that include the flexor pollicis longus, the index and long fingers of the flexor digitorum profundus, and the pronator quadrates. The classic site of anterior interosseous syndrome is an inability to perform the "OK" sign by approximating the tip of the index finger to the tin of the thumb. 3) The ulnar nerve innervates the flexor carpi Ulnaris and the medial half (i.e., digits 4-5) of the flexor digitorum profundus. The deep (motor) branch of the ulnar nerve innervates the hypothenar muscles, the interossei muscles, the two medial lumbricals, the adductor pollicis muscle, and the deep head of the flexor pollicis brevis. Weakness of the adductor pollicis and a positive Froment's sign are typical signs of an ulnar nerve palsy. 4) The posterior interosseous nerve is a deep (motor) branch of the radial nerve that innervates the following muscles: supinator, extensor carpi radialis brevis, extensor carpi Ulnaris, extensor digitorum, extensor indicis, extensor digiti Minimi, abductor pollicis longus, extensor pollicis longus, and extensor pollicis brevis. A palsy of the posterior interosseous nerve typically affects the ability to extend the fingers and the thumb.

• A patient was administered a narcotic medication one hour prior to the onset of their physical therapy session. What method of administration ensures that the medication will be 100 percent bioavailable? o 1. Intravenous injection o 2. Oral o 3. Inhalation o 4. Sublingual

o Answer: 1 • Bioavailability refers to the percentage of a drug that ultimately reaches the systemic circulation. The method of drug administration affects bioavailability since the drug must diffuse across cell membrane barriers and may also experience liver metabolism o Rationale: 1. Intravenous injection is the only method of drug administration that ensures 100% bioavailability. By injecting the drug directly into the systemic circulation, the drug is fully available to the bloodstream and thus the target tissues. 2. A drug administered orally would not be 100% bioavailable. The drug will travel down the gastrointestinal tract and must be absorbed through the intestines to reach the bloodstream. The entire dose of the drug may not pass through the intestinal wall. Even if the drug does pass through the intestinal wall, it then must travel through the liver (known as the first-pass effect). In the liver, the drug may experience significant metabolization before reaching the systemic circulation. 3. A drug administered via inhalation must diffuse across the alveoli where it then enters the pulmonary circulation. From the pulmonary circulation, it is transported directly to the heart and enters the systemic circulation. Though this form of administration avoids the first-pass effect of the liver. the drug still needs to be absorbed across the alveolar membrane and therefore is not 100% bioavailable. Additionally, drug particles may be trapped by cilia and mucus within the respiratory tract and not even reach the alveoli 4. A drug administered sublingually is absorbed through the oral mucosa into the venous system. The veins carry the drug to the heart where it can then enter the systemic circulation. Though sublingual administration avoids the first-pass effect of the liver, it still needs to be absorbed through the oral mucosa and is therefore not considered to be 100% bioavailable.

• A physical therapist treats a patient who has multiple myeloma. Which of the following systems should the therapist expect to be targeted once symptoms of the disease begin? o 1. Musculoskeletal o 2. Cardiovascular o 3. Integumentary o 4. Neurologic

o Answer: 1 • Multiple myeloma is a primary malignant cancer of plasma cells within the bone marrow. Initially, the cancer affects the bones and marrow of the vertebrae, ribs, skull, pelvis, and femur. The median age of diagnosis is approximately 70 ears of age. Although it affects multiple systems with its progression, it initially affects the musculoskeletal system and has potential for severe and devastating effects. o Rationale: 1. A patient recently diagnosed with multiple myeloma will typically experience a variety of symptoms affecting the musculoskeletal system such as skeletal muscle wasting, fatigue, and bone pain. Low-level exercise, fall prevention, and weight bearing activities as tolerated can assist the patient to manage the symptoms of this disease process. 2. The cardiovascular system is not typically a primary target of multiple myeloma. The patient will often experience fatigue, but this is secondary to the effects of the disease process rather than a compromised cardiovascular system. 3. The integumentary system is not typically a primary target of multiple myeloma. The patient may have subsequent skin breakdown in later stages of the disease process, especially if the patient becomes immobile. 4. The neurologic system is not typically a primary target of multiple myeloma. Neurologic complications can occur with progression of the cancer. The excessive bone loss can result in nerve compression as well as instability and damage to the spinal cord.

• A physical therapist reviews the medical record of a patient who sustained a spinal cord injury. A note recently entered by the physician indicates that the patient contracted a respiratory infection. Which type of spinal cord injury would be the MOST susceptible to this condition? o 1. Complete C4 tetraplegia o 2. Cauda equina lesion o 3. Brown-Sequard's syndrome o 4. Posterior cord syndrome

o Answer: 1 A patient with complete C4 tetraplegia will present with a loss of motor and sensory function secondary to damage to the spinal cord. Since the primary muscle of respiration, the diaphragm (C3-C5), is impaired, the patient will be unable to voluntarily or effectively ventilate. o Explanation: 1: A patient with complete C4 tetraplegia will have a reduced ventilator capacity due to muscle paralysis. The patient will exhibit limited ability to clear secretions, impaired chest mobility, and alveolar hypoventilation. 2: A cauda equina lesion is an injury that occurs below the L1 spinal level where the long nerve roots transcend. Cauda equina injuries are frequently incomplete due to the large number of nerve roots in the area and, as a result, are often considered to be peripheral nerve injuries. Characteristics include flaccidity, areflexia, and impairment of bowel and bladder function. 3: Brown-Sequard's syndrome is an incomplete lesion usually caused b a stab wound. which produces hemi section of the spinal cord. There is paralysis and loss of vibratory and position sense on the same side as the lesion due to the damage to the corticospinal tract and dorsal columns There is a loss of pain and temperature sense on the opposite side of the lesion from damage to the lateral spinothalamic tract. 4: Posterior cord syndrome is an extremely rare condition that presents with a loss of proprioception. two-point discrimination, graphesthesia, and stereognosis below the level of the lesion. Patients typical present with a wide-based steppage gait.

• A patient with suspected pulmonary embolism is referred for additional diagnostic testing. Which of the following patients would be at the GREATEST risk for developing this medical condition? o 1. A 35-year-old female who is on bed rest, obese, and on oral contraceptives o 2. A 45-vear-old male two weeks post tibial fracture requiring internal fixation o 3. A 70-vear-old female three weeks post closed reduction of a Colles' fracture o 4. A 50-vear-old male three weeks post malleolar fracture in a walking boot

o Answer: 1 Any condition that slows or changes the flow of blood within a vein can increase the risk for thrombus formation and ultimately a pulmonary embolism. These risk factors include prolonged immobility, recent surgery, pregnancy, obesity, cigarette smoking, medications (e.g., birth control, hormone replacement), blood conditions characterized by "thick" blood (e.g. polycythemia), blood clotting disorders, advanced age, cancer, and certain autoimmune disorders. o Explanation: 1. A 35-year-old obese female who is on bed rest and on oral contraceptives has the greatest number of risk factors for pulmonary embolism of the presented options. The risk factors in this scenario include obesity, prolonged immobility, and taking oral contraceptives. 2. A 45-year-old male two weeks post tibial fracture requiring internal fixation is at a relatively low risk for pulmonary embolism since he only has one risk factor (i.e., recent surgery). The risk for pulmonary embolism is greatest two to ten days post surgery. 3. A 70-vear-old female three weeks post closed reduction of a Colles' fracture is at a relatively low risk for pulmonary embolism. Although the risk for deep vein thrombosis increases with age (i.e., greater than 60), deep vein thrombosis is far more common in lower extremity fractures requiring surgical intervention 4. A 50-year-old male three weeks post malleolar fracture in a walking boot is at a relatively low risk for pulmonary embolism since the patient would most likely be ambulatory with an assistive device that allows non-weight bearing or partial weight bearing, depending on the restrictions set by the physician. Additionally, the patient would be allowed to remove the walking boot when not weight bearing and perform gentle range of motion of the ankle.

• A physical therapist treats a patient using a vacuum-assisted closure device. Which of the following BEST describes the mechanism used by this type of device to promote wound healing? o 1. Positive pressure in the wound bed o 2 Negative pressure in the wound bed o 3. Atmospheric pressure in the wound bed o 4. Alternating pressure in the wound bed

o Answer: 2 The use of vacuum-assisted closure devices is most commonly referred to as negative wound pressure therapy (NWPT). The specialized dressing and battery-powered vacuum device create the negative pressure environment. o Explanation: 1. Positive pressure refers to a state of increased pressure within a system or area relative to the pressure of the surrounding environment. Positive pressure would have the tendency to cause ischemia and discomfort that would impede healing. 2. Vacuum-assisted devices utilize a negative wound pressure environment that aids in the management of wound exudate and interstitial edema and facilitates growth of granulation tissue. 3. Atmospheric pressure refers to the normal pressure exerted on a wound by the surrounding air. 4. Alternating pressure is often utilized in specialized mattresses and overlays to prevent skin breakdown in individuals at risk for pressure injuries.

• A patient diagnosed with peripheral vascular disease begins a supervised treadmill exercise regimen intended to reduce claudication symptoms. After two minutes of ambulation, the patient reports the onset of claudication symptoms rated as 3/10 on a pain scale. What is the physical therapist's MOST appropriate response? o 1. Encourage the patient to continue until symptoms become more painful o 2. Reduce the speed and grade of the treadmill until symptoms resolve o 3. Allow the patient to rest, resuming ambulation once symptoms have subsided o 4 Discontinue use of the treadmill and select an alternative mode of exercise

o Answer: 1 Symptom provoking ambulation programs have been shown to effectively increase the intensity and duration of ambulation that patients with peripheral vascular disease (PVD) are able to tolerate before the onset of limiting claudication symptoms o Rationale: 1. Patients engaging in exercise training to decrease claudication symptoms should be encouraged to continue ambulating until symptoms increase to a moderate level of severity. Rest breaks should be initiated before symptoms approach an intolerable intensity or at the patient's request. A rating of 3/10 on a pain scale is relatively mild, and in the absence of other symptoms provides ample evidence to continue ambulation. 2. During initial training, speed and grade settings are typically selected with the intent of eliciting the patient's claudication symptoms within three to five minutes. An exercise-rest-exercise pattern of activity is recommended with exercise intensity remaining high enough to exacerbate symptoms. 3. Once symptoms become limiting with each exercise effort, the patient should rest in either sitting or standing until symptoms subside. The exercise-rest-exercise pattern should continue for at least 30 minutes with exercise intensity sufficient to elicit symptoms and require intermittent rest breaks. 4. Walking exercise regimens intended to elicit claudication symptoms have been shown to be an extremely effective mode of exercise for the treatment of PVD and claudication. A treadmill offers a consistent means of altering exercise parameters (e.g., speed, grade).

• A physical therapist evaluates a patient using the Modified Ashworth Scale and documents a rating of 3. Which of the following conditions would MOST likely result in a patient exhibiting a rating of 3 on this scale? o 1. Cerebral palsy o 2. Down syndrome o 3. Duchenne muscular dystrophy o 4. Guillain-Barre syndrome

o Answer: 1 The Modified Ashworth Scale (MAS) is a widely used qualitative scale for the assessment of spasticity and measures the amount of resistance to passive stretch. Spasticity is a common clinical manifestation of an upper motor neuron lesion. The scale ranges from O (no increase in muscle tone) to 4 (affected limb rigid in flexion or extension). A rating of 3 on the MAS is consistent with a considerable increase in muscle tone and difficulty with passive movement. o Explanation: 1. Cerebral palsy is an umbrella term used to describe a group of non-progressive movement disorders that result from brain damage. The clinical presentation is highly variable based on the area and extent of central nervous system damage. Spasticity is commonly associated with cerebral palsy and often presents in a manner consistent with a rating of 3 on the MAS 2. Down syndrome (trisomy 21) is a genetic abnormality consisting of an extra 21st chromosome. Signs and symptoms of this syndrome may include hypotonia, joint hypermobility, intellectual disability, flattened nasal bridge, narrow eyelids with epicanthal folds, flat feet, scoliosis, congenital heart disease, and visual and hearing loss. Since hypotonia rather than spasticity would be expected in a patient with Down syndrome, use of the MAS would be unlikely. 3. Duchenne muscular dystrophy (DMD) is a progressive neuromuscular degenerative disorder that manifests symptoms once fat and connective tissue begin to replace muscle that has been destroyed by the disease process. MD results in muscle weakness in a proximal to distal progression. The progression is usually rapid with the inability to ambulate by ten to twelve years of age. Since MD is a lower motor neuron disease and spasticity is not part of the clinical presentation. use of the MAS would be unlikely. 4. Guillain-Barre syndrome is a temporary inflammation and demyelination of the myelin sheaths of peripheral nerves. Guillain-Barre syndrome results in motor weakness in a distal to proximal progression, sensor impairment, and possible respirator paralysis. Since Guillain-Barre syndrome is a lower motor neuron disease and spasticity is not part of the clinical presentation, use of the MAS would be unlikely.

• A physical therapist asks a patient several questions prior to administering cervical mechanical traction. During the questioning, the patient indicates they are wearing dentures and that the dentures are securely in place. What is the MOST appropriate therapist action? o 1. Initiate traction using normal treatment parameters with the dentures in place o 2. Initiate traction using normal treatment parameters after removing the dentures o 3. Initiate traction using fifty percent of the normal recommended force with the dentures in place o 4. Avoid using traction since the intervention is contraindicated for the patient

o Answer: 1 The application of spinal traction is contraindicated in some circumstances (e.g., hypermobile joints, acute inflammation). In other cases, spinal traction should be applied with extra caution (i.e., precautions). o Explanation: 1. A patient who wears dentures should be instructed to keep the dentures in place during cervical traction. If the dentures are removed, the alignment of the temporomandibular joints (TMJ) may be altered, causing problems if pressure is applied through the mandible. 2. Removing the dentures would be inappropriate since this action could result in an alteration in alignment of the TMJs, causing problems if pressure is applied to these joints through the mandible 3. The proper protocol is to keep the dentures in place during treatment. therefore, treatment parameters do not need to be adjusted in the presence of dentures. Additionally, decreasing the force of traction would diminish the effectiveness of the treatment. 4. The presence of dentures is not a contraindication for cervical traction. It is however a precaution for treatment. If a patient normally wears dentures, the should remain in place during treatment.

• Following an acute myocardial infarction, a patient continues to experience frequent arrhythmias. A physical therapist should recognize an abnormality in which of the following lab values as being the MOST likely to contribute to the arrhythmias? o 1. Potassium o 2. Hemoglobin o 3. Hematocrit o 4. Platelet count

o Answer: 1 • A cardiac arrhythmia is a disturbance in heart rate and rhythm as a result of an abnormality in the electrical conduction system in the heart. Dysrhythmias are common complications of a myocardial infarction because of the interruption to the cardiac conduction system and imbalance of autonomic regulation. The severity and type of arrhythmia is dependent on the location and extent of damage to the myocardium. o Rationale: 1. Potassium plays a role in the generation of the cardiac action potential, specifically affecting the duration and repolarization. Both hypokalemia and hyperkalemia can result in the interruption of the action potential and cardiac dysrhythmias. Patients with hypokalemia may report dizziness and/or palpitations. 2. Hemoglobin is the oxygen carrying protein in red blood cells. Normal hemoglobin values for adult males are 13.3-16.2 g/dL and for females are 12.0-15.8 qm/dL Abnormalities in hemoglobin are associated with impaired aerobic capacity, diminished exercise tolerance, increased fatigue, and tachycardia. Therapeutic intervention is contraindicated with hemoglobin levels less than 8 qm/dL. 3. Hematocrit is the percentage of red blood cells in total blood volume. An elevated hematocrit level can occur with dehydration, polycythemia vera or an overproduction of red blood cells. Low hematocrit levels can indicate anemia or blood loss. Exercise is contraindicated with levels less than 25%. 4. The role of platelets is to initiate the clotting sequence to repair damaged blood vessels. Bleeding can occur with platelet levels less than 15.000-20.000 cells/mm3 With levels below 20.000 cells/mm3 activity should be limited to activities of daily living.

• A physical therapist treats a patient with multiple sclerosis. The patient does not have a history of seizures, however, has been prescribed an anticonvulsant medication to assist in the treatment of neuropathic pain. What would this form of treatment be BEST described as? o 1. Adjuvant medication o 2. Immunomodulating medication o 3. Interventional medicine o 4. Alternative medicine

o Answer: 1 • Anticonvulsant medications such as gabapentin and carbamazepine have been found effective with the treatment of neuropathic pain. Neuropathic pain is associated with injury to the peripheral or central nervous system. Common diagnoses associated with neuropathic pain include multiple sclerosis, diabetes mellitus, shingles, trauma, and cancer. The most common side effects of anticonvulsant medications are drowsiness, dizziness, and balance problems o Rationale: 1. The definition of adjuvant is "serving to aid, assist or contribute." Anticonvulsant medication used to treat neuropathic pain is an example of adjuvant medication. Adjuvant medication is a drug that is not typically used to treat pain (e.g., anticonvulsants), but is routinely prescribed for effective pain management within certain populations. Anticonvulsant and antidepressant medications are commonly used adjuvant medications for patients with multiple sclerosis. 2. Immunomodulating medication is a term used to describe a medication that is prescribed to assist with modifying the body's immune responses. Immunosuppressants and immunostimulants do not assist in the treatment of neuropathic pain in patients with multiple sclerosis. 3. Interventional medicine is a subspecialty of pain management that attempts to manage pain through the use of invasive techniques such as facet joint injections, nerve blocks, neuroaugmentation, and implantable drug delivery systems. Interventional medicine does not include the use of anticonvulsant medication to treat neuropathic pain in patients with multiple sclerosis. 4. Alternative medicine is a form of medical treatment that uses methods that are different from the typical Western scientific methods. Alternative medicine may use homeopathy or acupuncture to treat pain, however, would not include the use of anticonvulsant medication to treat neuropathic pain in patients with multiple sclerosis.

• A physical therapist examines a child with spastic quadriplegia cerebral palsy in the school setting. Based on this diagnosis, which muscle is MOST likely to be shortened? o 1. Flexor digitorum profundus o 2. Triceps o 3. Gluteus minimus o 4. Tibialis anterior

o Answer: 1 • Cerebral palsy is a non-progressive disorder of the central nervous system resulting in impairments in posture and volitional movement. Cerebral palsy is an umbrella term used to describe movement disorders due to brain damage acquired in utero, during birth or during infancy. It is classified by extremity involvement and muscle tone. Spastic quadriplegia describes an upper motor neuron lesion in the motor cortex of the cerebrum involving the trunk and all four extremities. o Rationale: 1. The flexor digitorum profundus produces flexion of the wrist and finger joints. With cerebral palsy, the wrist and finger flexors tend to demonstrate increased spasticity and are therefore shortened 2. The triceps produce extension of the elbow. With cerebral pals, the elbow flexors tend to demonstrate increased spasticity, not the elbow extensors 3. The gluteus minimus produces abduction of the hip. With cerebral palsy, the hip adductors tend to demonstrate increased spasticity, not the hip abductors. 4. The tibialis anterior produces dorsiflexion of the ankle. With cerebral palsy, the ankle plantar flexors tend to demonstrate increased spasticity. not the ankle dorsiflexors.

• A patient complains of extreme fatigue that prevented them from ambulating following a physical therapy session in which neuromuscular electrical stimulation was used on a lower extremity muscle. Which modification to the on: off time would be MOST appropriate based on the patient's complaint? o 1. Decreased on time: increased off time o 2. Decreased on time: decreased off time o 3. Increased on time; increased off time o 4. Increased on time; decreased off time

o Answer: 1 • Neuromuscular electrical stimulation (NMES) is a technique used to facilitate skeletal muscle activity. The intervention administers an electrical stimulus of appropriate intensity and duration to a peripheral nerve. NMES is a commonly used therapeutic technique to facilitate the return of controlled functional muscular activity. The on: off ratio specifies the relative amount of time the electrical stimulation is "on" versus the amount of time it is "off." The ideal on: off ratio with NMES is dictated by the established therapeutic objectives, although on time should be less than off time in order to prevent excessive muscle fatigue. o Rationale: 1. If a patient is complaining of excessive muscle fatigue following NMES, it is appropriate to decrease on time and increase off time. This results in a shorter period of muscle contraction and a longer rest period. This change in the NES parameters should result in diminished fatigue following the session 2. While on time should be decreased if a patient is experiencing excessive fatigue following a session of NMES, decreasing off time would result in a shorter rest period. Decreasing off time would not be an appropriate modification. The diminished off time may negate the benefit of decreasing the on time. 3. During an NMES treatment, on time signifies the time that the targeted muscle is contracting. It would not be appropriate to increase on time for a patient who is complaining of excessive fatigue. However, it would be appropriate to increase the off time for this patient. 4. It would not be appropriate to increase the on time for a patient complaining of fatigue since this action increases the time that the targeted muscle is contracting. It is not appropriate to decrease off time for this patient since this action decreases the rest period.

• A patient with hip joint pain is referred to physical therapy for direction in weight bearing strengthening exercises. The physician's referral notes that recent bone density testing revealed the patient has osteopenia. Which of the following comorbidities would MOST likely be part of the patient's medical history? o 1. Cushing's syndrome o 2. Guillain-Barre syndrome o 3. Spinal stenosis o 4. Complex regional pain syndrome

o Answer: 1 • Osteopenia is a condition presenting with low bone mass that is not severe enough to qualify as osteoporosis. Bone mineral density is used to diagnose osteoporosis and other low bone mass disorders. Individuals with osteopenia may not have actual bone loss, but a naturally lower bone density than established norms. A decrease in bone mass may be caused by prolonged drug therapies of heparin or corticosteroid use, endocrine disorders. malnutrition, and other disease processes o Rationale: 1. Cushing's syndrome is a condition resulting from abnormally high levels of cortisol due to endogenous overproduction of cortisol or excessive exogenous use of corticosteroids. High levels of cortisol in the body for a prolonged period changes the way in which bones are remodeled (i.e., decreased bone formation, increased bone resorption). These changes ultimately lead to osteopenia or osteoporosis for many of these patients. 2. Guillain-Barre syndrome (GBS) is a temporary inflammation and demyelination of the peripheral nerves' myelin sheaths, potentially resulting in axonal degeneration. GBS results in motor weakness in a distal to proximal progression, sensory impairment, and possible respiratory paralysis. The effects of GBS are temporary, with most patients experiencing a full recovery, and would not lead to a decrease in the patient's bone mass 3. Spinal stenosis refers to a narrowing of either the vertebral or intervertebral foramina. Symptoms are typically produced as a result of mechanical compression on either the spinal cord or exiting nerve roots and may include pain, paresthesia, weakness, and diminished reflexes. Spinal stenosis is not associated with a diagnosis of osteopenia 4. Complex regional pain syndrome is characterized b an increase in sympathetic activity causing a release of norepinephrine in the periphery and subsequent vasoconstriction of blood vessels resulting in pain and an increase in sensitivity to peripheral stimulation. Complex regional pain syndrome is not associated with a diagnosis of osteopenia.

• A physical therapist reviews the medical record of a patient admitted to the hospital with suspected renal involvement. Which laboratory test would be the MOST useful to assess the patient's present renal function? o 1. Platelet count o 2. Hemoglobin o 3. Blood urea nitrogen o 4. Hematocrit

o Answer: 3 Blood urea nitrogen is a common measure used to assess renal function. The normal blood urea nitrogen level for adults is 10-20 mg/dL. o Explanation: 1. Platelet count identifies the number of platelets present in whole blood. If the platelet level is high, it indicates increased risk of thrombosis and if the level is low. It indicates increased risk of bruising and bleeding. 2. Hemoglobin is the iron containing pigment in red blood cells that functions to carry oxygen in the blood. Low hemoglobin may indicate anemia or blood loss. Elevated hemoglobin suggests polycythemia or dehydration 3. Blood urea nitrogen is used to assess kidney function. An increased blood urea nitrogen level can be indicative of dehydration, renal failure or heart failure. A decreased blood urea nitrogen level can be indication of malnourishment, hepatic failure or pregnancy. 4. Hematocrit measures the percentage of red blood cells in a volume of blood. Hematocrit may be decreased with anemia, nutritional deficiency, and leukemia. Hematocrit may be increased with dehydration, polycythemia, and burns

• A patient two weeks status post anterior cruciate ligament reconstruction is referred to physical therapy for neuromuscular reeducation using an electrical stimulation unit. The physical therapist has the patient perform quad sets using the electrical stimulation unit at home. What is the MOST beneficial frequency of treatment to promote muscle reeducation in this scenario? o 1. Two times per day o 2. Three times per week o 3. One time per week o 4. Once every two weeks

o Answer: 2 • Neuromuscular reeducation is a common focus of treatment following a surgery or injury that may result in muscular inhibition. Frequency is a variable that should be considered when prescribing neuromuscular reeducation activities, as well as intensity, repetitions, and sets o Rationale: 1. When performing neuromuscular reeducation exercise, the exercise can be repeated several times per day. Since the intensity of the exercise (i.e., quad sets) is low, the patient would not likely have a negative response to performing it multiple times daily. 2. Three times per week is a general guideline for strengthening activities, though it would not be sufficient when performing neuromuscular reeducation exercise. Neuromuscular reeducation exercise is typically performed multiple times a day, especially when the intensity of the given exercise is low. 3. One time per week is insufficient based on the stated goal of promoting neuromuscular reeducation. This frequency may be more appropriate for a general strengthening maintenance program. 4. Once every two weeks is insufficient for virtually any therapeutic purpose, most notably neuromuscular reeducation

• A physical therapist works with a patient with hemiparesis who uses a hemiplegic chair for mobility. Which activity would become more challenging for the patient based on this specific type of wheelchair? o 1. Reaching for objects outside the base of support o 2. Standing up from the seat of the chair o 3. Performing independent pressure relief o 4. Elevating the legs for edema management

o Answer: 2 A hemiplegic chair incorporates a seat that is approximately two inches lower than a standard chair to enable the user to use the lower extremities to propel the chair. The patient typically uses one handrim and one or both feet to help propel and steer the wheelchair. One or both front riggings on the wheelchair are removed to provide the feet with necessary space for propulsion. o Explanation: 1. Reaching for objects outside the base of support may be challenging for the patient depending on the extent of involvement. However, a hemiplegic chair would not increase the complexity of this task. 2. Standing up from the seat of the chair would be more challenging with a hemiplegic chair since the lower seat would require significantly more upper and lower extremity strength to attain a standing position. 3. Performing independent pressure relief may be challenging for the patient depending on the extent of involvement. However, a hemiplegic chair would not increase the complexity of this task. 4. Elevating leg rests is not a feature that is specific to a hemiplegic chair. If elevating leg rests were required for this patient, a hemiplegic chair would not increase the complexity of this task.

• A patient recovering from pneumonia is prescribed a pharmacological agent administered with a nebulizer. Which pharmacological agent was MOST likely utilized? o 1. Calcium channel blocker agents o 2. Mucolytic agents o 3. Nitrate agents o 4. Sedative hypnotic agents

o Answer: 2 A nebulizer is a device commonly used to administer medication in the form of a mist that is inhaled into the lungs. Patients recovering from pneumonia often present with viscous mucus secretions that inhibit proper lung functioning. o Explanation: 1. Calcium channel blocker agents decrease the entry of calcium into vascular smooth muscle cells resulting in diminished myocardial contraction. vasodilation, and decreased oxygen demand of the heart. They are used to treat hypertension, angina pectoris, arrhythmias, and congestive heart failure. Calcium channel blocker agents are typically administered orally. 2. Mucolytic agents decrease the viscosity of mucus secretions by altering their composition and consistency, making them easier to expectorate. They are used to treat pneumonia, emphysema, chronic bronchitis, and cystic fibrosis. Mucolytic agents are typically administered by a nebulizer. 3. Nitrate agents decrease ischemia through smooth muscle relaxation and dilation of peripheral vessels. The are used to treat angina pectoris. Nitrate agents are typically administered sublingually. 4. Sedative agents produce a calming effect and relaxation, while hypnotic agents induce sleep. Sedative agents are used to treat anxiety and insomnia and for preoperative sedation. They are typically administered orally.

• A physical therapist examines normative values for lung volumes prior to performing pulmonary function testing on a patient. Which of the following lung volumes would typically have the GREATEST value? o 1. Tidal volume o 2. Inspiratory reserve volume o 3. Expiratory reserve volume o 4. Residual volume

o Answer: 2 A physical therapist should have knowledge of normal lung volumes and capacities. On average, a healthy adult male has a total lung capacity of approximately 6000 mL, while a female has a total lung capacity of 4200 mL. o Explanation: 1. Tidal volume (TV) is the amount of air inspired and expired with each breath during quiet breathing and it accounts for approximately 10 percent of total lung volume. TV for males is typically 600 mL, while TV for females is typically 500 ml 2. Inspiratory reserve volume (IRV) is the maximal volume of air that can be inspired after normal tidal inspiration and it accounts for approximately 50 percent of total lung volume. IRV for males is typically 3000 mL, while IRV for females is tvoicallv 1900 mL. 3. Expiratory reserve volume (ERV) is the maximal volume of air that can be exhaled after a normal tidal exhalation and it accounts for approximately 15 percent of total lung volume. ERV for males is typically 1200 mL. while ERV for females is typically 800 mL. 4. Residual volume (RV) is the volume of gas remaining in the lungs at the end of a maximal expiration and it accounts for approximately 25 percent of total lung volume. RV for males is typically 1200 mL, while RV for females is typically 1000 ml

• A physical therapist performs prosthetic training with a patient post transfemoral amputation. What initial instruction would be the MOST appropriate when ascending the stairs? o 1. Utilize the handrail to propel your legs to the next step simultaneously o 2. Place your body weight on the prosthetic side and lead with your uninvolved leg o 3. Place your body weight on the uninvolved side and lead with our prosthesis o 4. Avoid using stairs with your prosthesis

o Answer: 2 • A patient with a unilateral transfemoral amputation will ascend stairs leading with the uninvolved lower extremity. This allows for greater stability as the uninvolved lower extremity uses its strength to lift the patient to the next step with the prosthetic side to follow. o Rationale: 1. A handrail will assist a patient when ascending and descending the stairs, however, the patient should not rely on a handrail and upper extremity strength to advance up or down a step or flight of stairs. 2. This sequence of ascending with the uninvolved lower extremity is used for any unilateral weakness in order to have the uninvolved lower extremity lift the body weight against gravity to the next step. 3. The patient would not effectively ascend the stairs with the prosthetic side secondary to lack of proprioception, sensation, strength, and control of the knee joint of the prosthesis. 4. A patient should not have to avoid stairs secondary to having a prosthetic limb. A patient with a transfemoral prosthesis should be able to ambulate on all surfaces with or without an assistive device

• A patient reports being unable to drive due to difficulty reaching forward to grasp the steering wheel secondary to weakness, though they deny the presence of pain. Which of the following conditions would MOST likely be associated with this toe of activity limitation? o 1. Rotator cuff impingement o 2. Peripheral nerve entrapment o 3. Adhesive capsulitis o 4. Cervical facet impingement

o Answer: 2 • In the upper extremity, brachial plexus and other peripheral. nerve injuries can result from trauma, penetration, traction on compression. The three classifications of nerve injury in order of severity (from least to most severe) are neurapraxia, axonotmesis, and neurotmesis. The degree of injury is related to the severity and duration of compression (i.e.. entrapment). Symptoms include paresthesia's, pain, and weakness depending on the severity of the nerve injury o Rationale: 1. Rotator cuff impingement is caused by the humeral head and the associated rotator cuff attachments migrating proximally and becoming impinged on the undersurface of 2. Peripheral nerve entrapment is the most likely of the options to impact the ability to drive due to weakness causing an inability to reach forward and grasp the steering wheel. Peripheral nerve entrapment can cause wasting of muscles and weakness. as well as diminished or absent sensation to the area supplied by the nerve, depending on the severity of the iniurv. 3. Adhesive capsulitis ("frozen shoulder") is characterized by inflammation and fibrotic thickening of the anterior capsule of the shoulder. Loss of motion occurs in a capsular pattern with the greatest restriction of motion being lateral rotation, then abduction, then medial rotation. Difficulty reaching for the steering wheel would more likely be due to pain and/or limited motion with adhesive capsulitis rather than weakness. 4. Cervical facet impingement ("facet syndrome") is characterized by inflammation and pain in one or more of the facet joints of the cervical spine. The facet (zygapophyseal) joints are the true synovial intervertebral joints with a joint capsule that can become impinged. Due to the natural coupling motion in the spine, painful and limited rotation and lateral flexion of the neck to the same side is typically seen in cervical facet impingement. Although driving may be difficult, it would more likely be due to difficult turning the head when driving rather than reaching forward to grasp the steering wheel.

• A physical therapist examines a patient with an acute burn to the left shoulder and axilla area secondary to a spill of boiling water. What is the MOST appropriate splint for this patient? o Shoulder sling o Airplane splint o C-bar splint o Hemi-arm sling

o Answer: 2 • Patients with burns to the shoulder and axilla area will tend to develop contractures toward shoulder adduction and internal rotation. As a result. the motions of shoulder abduction. flexion, and external rotation should be emphasized in physical therapy and with splinting. o Rationale: 1. A shoulder sling is used for shoulder pain or subluxation in order to protect and immobilize the shoulder joint. Immobilization promotes tissue healing and prevents overstretching of muscles or ligaments. Typically when using this type of sling, the shoulder is positioned in slight abduction and internal rotation with the elbow in flexion. 2. An airplane splint is a type of complex splint that holds the arm in abduction with the elbow in approximately 90 degrees of flexion primarily protecting against a shoulder adduction contracture. Typically, this type of splint is used by patients with burns affecting the shoulder and axillary regions. 3. A c-bar splint is a type of a hand-finger splint used to promote thumb abduction and opposition. The splint facilitates improved hand function when weakness exists. This type of splint can be used following surgery to promote tissue healing and provide immobilization of the thumb. 4. A hemi-arm sling is a type of vertical arm sling that uses a humeral cuff with figure-eight suspension to provide a vertical upward force that supports the humerus without restricting the elbow and forearm. This type of sling is typically used with patients with shoulder subluxation.

• A patient is admitted to an acute care hospital following a motor vehicle accident. The patient is presently in a state of unconsciousness without arousal, eye opening or sleep-wake cycles. The patient does possess intact brainstem reflexes upon examination. Which diagnosis is MOST likely? o 1. Brain death o 2. Coma o 3. Vegetative state o 4. Locked-in syndrome

o Answer: 2 • The abnormal state of consciousness encompasses many terms such as clouding of consciousness, confusional state, delirium, lethargy, obtundation, stupor, dementia, hypersomnia, vegetative state, akinetic mutism, locked-in syndrome, coma, and brain death. o Rationale: 1. Brain death refers to loss of function of the entire cerebrum and brainstem, resulting in coma, no spontaneous respiration, and loss of all brainstem reflexes. Spinal reflexes, including deep tendon, plantar flexion, and withdrawal reflexes, may remain. Recovery does not occur. 2. Coma refers to unresponsiveness from which the patient cannot be aroused and the eves do not open in response to stimulation. A coma is considered a medical emergency in which medical intervention attempts to preserve life and brain function. Recover is dependent on the cause of the coma and the level of brain damage sustained. A coma typically results from trauma, swelling, bleeding, CVA, blood sugar irregularity, oxygen deprivation, infection, seizures, and toxins. 3. A vegetative state refers to the absence of responsiveness and awareness due to dysfunction of the cerebral hemispheres. There is preservation of autonomic and motor reflexes and sleep-wake cycles. Patients may perform eve movements, awning, and involuntary movements to noxious stimuli, but show no awareness of self or external stimuli. Recovery is dependent on cause and although patients can improve, they can also decline into persistent or permanent vegetative states. 4. Locked-in syndrome results from a lesion to the brainstem, most frequently an ischemic pontine lesion. It is characterized b preserved awareness, relatively intact cognitive functions, normal sleep-wake cycles, and the ability to communicate while being completely paralyzed and voiceless. Vertical eve movement is typically possible and often patients open and close their eyes or blink to answer questions.

• A physical therapist performs strength testing on members of a high school football team as part of a preseason assessment. The majority of the players are between the ages of 14 and 16 years of age. Which age-related phenomenon would NOT be characteristic of this period of development? o 1. Increase in muscle mass o 2. Increase in the number of muscle fibers o 3. Increase in muscle fiber size o 4. Increase in muscle strength

o Answer: 2 • The age of the football players is consistent with puberty (i.e. Adolescence). During this period of life, there is a rapid acceleration in a number of attributes related to strength and performance. o Rationale: 1. Muscle mass refers to the actual size of the muscle. An increase in muscle mass is characteristic of puberty primarily due to the hormonal influence of growth hormone and testosterone. During puberty, muscle mass can increase more than 30% per year. 2. The number of muscle fibers is determined prior to birth or very early in infancy. Although the number of muscle fibers does not change during puberty, significant strength gains occur due to the rapid acceleration in muscle fiber size and muscle mass. 3. The increase in muscle mass that occurs during puberty is caused bv an increase in the muscle fiber size. This occurs as a result of the muscle increasing in size and in the amount of contractile proteins which comprise the myofibrils within each muscle fiber. 4. Muscle strength increases dramatically during puberty, particularly in boys due to the hormonal differences between the sexes. Longitudinal studies have shown that muscle strength can increase approximately 30% per year in boys from the ages of 10 to 16.

• A physical therapist documents gait training performed during a treatment session. Which descriptive term is only associated with the swing phase of the gait cycle? o 1. Heel strike o 2. Deceleration o 3. Loading response o 4. Midstance

o Answer: 2 • The phases of gait are classified based on either points in time (traditional terminology) or periods of time (Rancho Los Amigos terminology). Stance phase represents approximately 60 percent of the gait cycle, while swing phase represents approximately 40 percent o Rationale: 1. Heel strike is traditional terminology that refers to the instant that the heel touches the ground to begin stance phase. Heel strike is a component of stance phase. 2. Deceleration is traditional terminology that begins directly after mid-swing as the swing limb begins to extend and ends just prior to heel strike. Deceleration is a component of swing phase. 3. Loading response is Rancho Los Amigos terminology that corresponds to the amount of time between initial contact and the beginning of the swing phase for the other leg. Loading response is a component of stance phase. 4. Midstance is a term utilized in traditional terminology and Rancho Los Amigos terminology. In traditional terminology, midstance refers to the point during the stance phase when the entire body weight is directly over the stance limb. In Rancho Los Amigos terminology, midstance corresponds to the point in the stance phase when the other foot is off the floor until the body is directly over the stance limb. Midstance is a component of stance phase

• A physical therapist orders a wheelchair with anti-tip tubes for a patient in preparation for discharge from a rehabilitation hospital. A patient with which of the following conditions would benefit the MOST from this option? o 1. Guillain-Barre syndrome o 2. C5 tetraplegia o 3. Stroke with hemiparesis o 4. Amyotrophic lateral sclerosis

o Answer: 2 • Wheelchairs can be manually propelled or externally powered. Considerations when selecting an appropriate wheelchair and its component options include the patient's physical needs, physical abilities, cognition, coordination, and endurance. Anti-tip tubes are an easy to install option that fit most standard models and are designed to instantly improve patient safety by preventing the wheelchair from tipping backwards. o Rationale: 1. Guillain-Barre syndrome (GBS) is a temporary inflammation and demyelination of the peripheral nerves' myelin sheaths. GBS results in motor weakness in a distal to proximal progression, sensory impairment, and possible respiratory paralysis. Since the extent of impairment for each patient depends on the clinical course of their condition, wheelchair needs will be highly variable. Anti-tip tubes would be more consistently needed for patients with spinal cord injuries. 2. Anti-tip tubes are commonly used with patients who have spinal cord injuries. In this particular case, a patient with C5 tetraplegia would have significant physical limitations and would be dependent on a wheelchair indefinitely for mobility needs. Anti-tip tunes provide the patient with a measure of safety in the event of backward tipping. 3. The clinical presentation of a cerebrovascular accident, commonly referred to as a stroke, is determined by the location and extent of the infarct. Since typical characteristics of a stroke include hemiplegia or hemiparesis, a hemi frame wheelchair or a one-hand drive frame would be the most likely needed components 4. Amyotrophic lateral sclerosis (ALS) is a chronic degenerative disease that produces both upper and lower motor neuron impairments. ALS is usually a rapidly progressing neurological disease with an average course of two to five years with 20-30% of patients surviving longer than five years. Patients with ALS will likely need frequent modifications to their wheelchair as the disease progresses and eventually will become dependent for all mobility. Due to the rapid progression of ALS and subsequent physical limitations, anti-tip tubes become a less critical wheelchair adaptation for this population.

• A patient with osteogenesis imperfect is examined by a physical therapist due to reports of upper extremity pain. The therapist decides to perform a series of special tests as part of the examination. Given the diagnosis, which of the following special tests would MOST likely be positive? o 1. Cervical foraminal compression test o 2. Adson maneuver o 3. Sulcus sign o 4. Lift off sign

o Answer: 3 • Osteogenesis imperfect is a connective tissue disorder that affects the formation of collagen during bone development. The clinical presentation includes pathological fractures, osteoporosis (i.e., brittle bones), hypermobile joints, bowing of the long bones, weakness, scoliosis, and impaired respiratory function. o Rationale 1. The cervical foraminal compression test is a special test used to detect the presence of cervical nerve root compression. A positive test is indicated by pain radiating into the arm when the therapist exerts a downward force through the patient's head with the neck in lateral flexion. A patient with osteogenesis imperfect is not at an increased risk for cervical nerve root compression. 2. The Adson maneuver is a special test used to detect thoracic outlet syndrome. A positive test is indicated by an absent or diminished radial pulse when the patient rotates their head to face the test shoulder and extends their head while the therapist laterally rotates and extends the patient's shoulder. A patient with osteogenesis imperfecta is not at an increased risk for thoracic outlet syndrome. 3. The sulcus sign is a special test used to detect inferior glenohumeral instability. A positive test is indicated by the presence of a depression seen between the acromion and humeral head (i.e., sulcus sign) when the therapist positions the patient's arm in 20-50 degrees of abduction and then pulls the arm inferiorly. Since patients with osteogenesis imperfect present with hypermobile joints. any special test designed to assess joint laxity will likely be positive. 4. The lift off sign is a special test used to detect the presence of a subscapularis lesion. A positive test is indicated if a patient is unable to lift the dorsum of their hand away from their low back. While patients with osteogenesis imperfect do exhibit muscular weakness. their level of weakness would not likely inhibit their ability to perform this test

• A physical therapist evaluates a three-year-old child recently diagnosed with a genetic disorder. The physical therapy plan of care includes monitoring the progression of muscle weakness, maintenance of range of motion and flexibility, and family education. Which diagnosis would MOST likely require this form of intervention? o 1. Down syndrome o 2. Cerebral palsv o 3. Duchenne muscular dystrophy o 4. Osteogenesis imperfecta

o Answer: 3 A physical therapist should understand underlying pathology of commonly encountered pediatric diagnoses in order to develop appropriate physical therapy care plans. Knowledge of short-term and lonq-term expectations for a given disease process will allow efficient. realistic, and proper care for each patient. o Explanation: 1. Down syndrome (trisomv 21) occurs when there is an error in cell division and the cell nucleus results in 47 chromosomes. The clinical presentation includes developmental delay, hypotonia, laxity of the ligaments, and poor strength. Interventions include strengthening and endurance activities. Down syndrome does not present with progressive weakness or loss of range of motion. 2. Cerebral palsv is an umbrella term used to describe a group of non-progressive movement disorders that result from brain damage. The clinical presentation includes abnormal muscle tone, impaired modulation of movement. presence of abnormal reflexes, and impaired mobility. Intervention includes ongoing family education, normalization of tone, stretching, strengthening, motor learning, and acquisition of developmental milestones. Cerebral palsy does not present with progressive weakness since it is a non-progressive disorder. 3. Duchenne muscular dystrophy is a progressive disorder caused by the absence of the gene required to produce the muscle proteins dystrophin and nebulin. Fat and connective tissue eventually replace muscle, and premature death usually occurs from cardiopulmonary failure. Symptoms include progressive weakness, falling, toe walking, excessive lordosis, and pseudohypertrophy of muscle groups. Physical therapy is initially indicated to acquire a baseline and determine the pattern and rate of disability. Intervention should focus on maintaining range of motion and flexibility. 4. Osteogenesis imperfect is an autosomal disorder of collagen synthesis that affects bone metabolism. Children with osteogenesis imperfect often have delayed developmental milestones secondary to ongoing fractures that result in immobilization, hypermobility of joints, and poorly developed muscles. Interventions include safe handling techniques, active range of motion emphasizing symmetrical movements, positioning, functional mobility, fracture management, and the use of orthoses. Osteogenesis imperfecta does not present with progressive weakness since it is a non-progressive disorder.

• A physician instructs a patient to utilize a knee derotation brace for all athletic activities. Which of the following conditions would MOST warrant the use of this type of brace? o 1. Knee osteoarthritis o 2. Anterior cruciate ligament reconstruction o 3. Anterior cruciate ligament insufficiency o 4. Posterior cruciate ligament reconstruction

o Answer: 3 Derotation braces are most effective in patients with ligamentous instability, usually involving the anterior and posterior cruciate ligaments. The literature is inconclusive on the efficacy of functional bracing following reconstruction. o Explanation: 1. Since derotation braces are most often used in the presence of ligamentous instability, a patient with knee osteoarthritis would not likely use this type of brace. A patient with knee osteoarthritis would be more likely to benefit from an unloader brace. 2. Anterior cruciate ligament reconstruction is typically performed due to disabling instability or frequent episodes of the knee giving way. The purpose of the surgical procedure is to reduce functional instability. Full return to vigorous activities following AC reconstruction often takes a minimum of four to six months. 3. A patient with anterior cruciate ligament insufficiency would be far more likely to experience functional instability than a patient who had anterior cruciate ligament reconstruction. As a result, the patient with the insufficiency would be a better candidate for the derotation brace. 4. Posterior cruciate ligament reconstruction is considerably less common than anterior cruciate ligament reconstruction. The purpose of the surgical procedure is to reduce functional instability. Full return to vigorous activities following PC reconstruction often takes nine months to one year.

• A physical therapist instructs a patient rehabilitating from a rotator cuff repair in a home exercise program. The patient is 27 years old and illiterate. What is the MOST appropriate action to promote compliance with the exercise program? o 1. Ask the patient to memorize the exercises o 2. Use short sentences consisting of simple words Physical therapists should try to keep exercise instructions as simple as possible, however, given the patient's illiteracy, this modification may still be inadequate to meet the patient's needs. o 3. Draw pictures to describe the exercises o 4. Utilizing a home exercise program

o Answer: 3 Illiterate refers to the inability to read or write simple sentences. Functional illiteracy refers to the inability of an individual to use reading, writing, and computational skills efficiently in everyday life situations. o Rationale: 1. Memorizing the exercises would be a formidable challenge for man patients. The absence of a handout for the patient to refer to would likely decrease compliance and may increase the probability of the exercises being performed incorrectly. 2. Physical therapists should try to keep exercise instructions as simple as possible, however, given the patient's illiteracy, this modification may still be inadequate to meet the patient's needs. 3. Pictures provide the patient with an image of the exercises without relying solely on formal written instructions or memorization. Home exercise programs are a critical component of almost any patient care plan and can be effective with patients that are illiterate. Physical therapists should use alternate forms of educational media (e.g., pictures) whenever possible since it is an erroneous assumption to believe that the vast majority of patients possess basic reading and writing skills.

• A physical therapist notices that a patient status post sternotomy has significantly altered their breathing pattern. The therapist is concerned that the alteration in breathing may lead to atelectasis. What is the MOST appropriate intervention to treat and prevent atelectasis? o 1. Active cycle of breathing o 2. Inspiratory muscle training o 3. Sustained maximal inspiration o 4. Postural drainage

o Answer: 3 • After a sternotomy, it may be painful for the patient to take a deep breath. Shallow breathing will lead to hypoventilation of the alveoli. which may cause atelectasis. In a sustained maximal inspiration, a maximal inspiratory effort is held at the point of maximum inspiration before exhalation. Prolonging and sustaining the maximum inspiration promotes recruitment of inspiratory muscle fibers and the drop in intrathoracic pressure enhances distribution of inhaled air. o Rationale: 1. The active cycle of breathing (ACB) technique was developed under the name "forced expiratory technique" and is most commonly used to assist secretion clearance in patients with asthma. It includes three phases: breathing control. thoracic expansion exercises, and forced expiratory technique. This technique would not be used to prevent atelectasis. 2. Inspiratory muscle training (IMT) uses a handheld breathing training device to provide resistance to increase the strength and endurance of the muscles of inspiration. IMT is not used to prevent atelectasis. 3. Prolonging and sustaining the maximum inspiration increases ventilation by promoting recruitment of inspiratory muscle fibers, promotes air passage past mucus obstructions in airways. maximizes alveolar expansion, and enhances distribution of inhaled air. Sustained maximal inspiration is the most appropriate intervention to use to prevent atelectasis after sternotomy. 4. Postural drainage consists of positioning the patient so that gravity will help drain bronchial secretions from specific lung segments toward the gravity-dependent central airways where they can be removed b cough or mechanical aspiration.

• A skilled nursing facility plans to expand a hallway to accommodate two wheelchairs passing simultaneously. In order to comply with the Americans with Disabilities Act standards, what is the required minimum width of the hallway? o 1. 36 inches o 2. 48 inches o 3. 60 inches o 4. 108 inches

o Answer: 3 • The Americans with Disabilities Act (ADA) is federal legislation that protects individuals with a physical or mental impairment that substantially limits one or more of their major life activities. This piece of legislation includes accessibility requirements for public spaces. o Rationale: 1. According to ADA standards, hallways should be 36 inches wide for one wheelchair to pass through. This width would not be sufficient for two wheelchairs to pass simultaneously. 2. The ADA requires that hallways are a minimum of 36 inches wide to allow a wheelchair to pass through. A width of 48 inches is wide enough for one wheelchair to pass, though it does not allow two wheelchairs to pass simultaneously. 3. The ADA requires that hallways are a minimum of 36 inches wide to allow a single wheelchair to pass through. According to the ADA, for two wheelchairs to pass simultaneously, a hallway should be a minimum of 60 inches. 4. The ADA requires that hallways are a minimum of 36 inches to allow a wheelchair to pass through. In order for two wheelchairs to pass simultaneously. a hallway should be a minimum of 60 inches. A hallway width of 108 inches is significantly greater than the minimum amount needed for two wheelchairs to pass

• A physical therapist examines a patient following a traumatic crush injury to the forearm. The patient displays a wrist drop deformity and is unable to strongly grasp objects with their hand. Which nerve was MOST likely affected? o 1. Median nerve o 2. Ulnar nerve o 4. 3. Radial nerve o Musculocutaneous nerve

o Answer: 3 • Wrist drop deformity is characterized b an inability to actively extend the wrist and fingers, resulting in the wrist being flaccidly held in a flexed position. Patients with wrist drop deformity have diminished grip strength since they are unable to actively extend their wrist. o Rationale: 1. The median nerve innervates the forearm pronators, most of the wrist flexors, first and second lumbricals, and muscles of the thenar eminence. Injury to the median nerve often causes weakness with wrist flexion and forearm pronation. 2. The ulnar nerve innervates the flexor carpi Ulnaris, flexor digitorum profundus, third and fourth lumbricals, muscles of the hypothenar eminence, interossei, and other small muscles within the hand. Injury to the ulnar nerve often causes weakness with wrist flexion and flexion of the ulnar half of the digits, resulting in a claw hand deformity. 3. The radial nerve innervates the triceps, brachioradialis, wrist extensors, and finger extensors. Iniurv to the radial nerve often causes weakness with wrist and finger extension, resulting in a wrist drop deformity. 4. The musculocutaneous nerve innervates the coracobrachialis, biceps brachii, and brachialis. Injury to the musculocutaneous nerve often causes weakness with elbow flexion and forearm supination.

• A physical therapist assesses a patient's muscle tone following a stroke and reports that while the patient has marked tone throughout the range of motion, their extremity can still be easily moved. Using the Modified Ashworth Scale, which grade of spasticity would be the MOST appropriate for this patient? o 1. 1 o 2. 1+ o 3. 2 o 4. 3

o Answer: 3. • The Modified Ashworth Scale (MAS) is an instrument that is used to assess muscle spasticity. The MAS uses ordinal scoring, with a grade of "O" indicating the absence of spasticity and a grade of "4" indicating the presence of rigidity. o Rationale: 1. A grade of "y" on the MAS is described as a slight increase in tone. This increase in tone may manifest as a "catch and release" or may be characterized by an increase in tone at the end of the range of motion. 2. A grade of "1+" on the MAS is described as a slight increase in tone. This grade differs from the previous grade in that the "catch" is followed by minimal resistance throughout the rest of the range of motion, which is usually less than half of the total range. 3. A grade of "2" on the MAS is described as a marked increase in tone throughout most of the range of motion. Despite the increase in tone, the affected part can still be easily moved through the range of motion. 4. A grade of "3" on the MAS is described as a considerable increase in tone. This grade differs from the previous grade in that the affected part is not easily moved through the range of motion.

• A patient in the physical therapy gym suddenly begins to cough, which is shortly followed by wheezing. The physical therapist, recognizing the possibility of an airway obstruction, should take which of the following actions? o 1. Attempt to ventilate o 2. Administer abdominal thrusts o 3. Perform a quick finger sweep of the mouth o 4. Continue to observe the patient, but do not interfere

o Answer: 4 Coughing and wheezing indicate that the airway is not completely obstructed. As a result. the physical therapist should continue to monitor the patient, however, should not formally intervene. Usually a patient that is coughing will independently dislodge the object causing the obstruction o Explanation: 1. If the patient is not breathing, a rescuer should open the airwav and attempt to ventilate. If the rescuer is unable to make the patient's chest rise, the rescuer should reposition and ventilate again. If the chest still does not rise, the rescuer must consider that there is an obstruction. This technique would not be appropriate for a person that is coughing or wheezing in an attempt to clear an obstruction. 2. If the patient is choking, the rescuer should attempt to perform the Heimlich maneuver while the conscious patient is in sitting or standing. The Heimlich maneuver attempts to remove the obstruction by providing abdominal thrusts. The rescuer would press the fist into the patient's abdomen with a quick inward and upward thrust with the intent of relieving the obstruction. This technique would not be appropriate for a person that is coughing or wheezing. 3. A rescuer should use a finger sweep only when thev can see solid material obstructing the airway of an unresponsive patient. If the rescuer were to do this without seeing the blockage, it may harm the patient or rescuer. This technique would not be appropriate for a person that is coughing or wheezing in an attempt to clear an obstruction. 4. If a mild obstruction is present and the patient is coughing or wheezing, the rescuer would not interfere with the patient's spontaneous coughing and breathing efforts. The rescuer should attempt to relieve the obstruction only if signs of severe obstruction develop such as the cough becoming silent, respiratory difficulty increasing or the patient becoming unresponsive.

• A physical therapist attempts to determine the relative increase in tissue temperature using ultrasound. Which parameters would result in the GREATEST amount of tissue heating? o 1. 1 MHz, 0.5 W/cm2 o 2. 1 MHz. 2.0 W/cm2 o 3. 3 MHz, 0.5 W/cm2 o 4. 3 MHz. 2.0 W/cm2

o Answer: 4 Factors that may affect the level of tissue heating when using ultrasound include the frequency, average intensity, duration, and tvoe of tissue to be heated. Therapists must determine the desired therapeutic effects for each patient and select parameters consistent with the intended effects. o Explanation: 1. Low frequency ultrasound (i.e., 1 MHz) has a greater depth of penetration, but a lower maximum temperature is achieved. Likewise, a lower ultrasound intensity (i.e., 0.5 W cm2) would result in a lower amount of tissue heating 2. Though a higher ultrasound intensity (i.e.. 2.0 W/cm2) would result in a higher amount of tissue heating, this is still not the best option since low frequency ultrasound results in a lower maximum temperature. 3. High frequency ultrasound (i.e., 3 MHz) does not penetrate as deep as low frequency ultrasound. However, a higher maximum temperature can be achieved. This option is still not the best option since the intensity used is low. 4. The greatest amount of tissue heating can be achieved with ultrasound by using a high frequency and a high intensity.

• A physical therapist identifies a bluish discoloration of the skin and nailbeds of a patient referred to physical therapy for pulmonary rehabilitation. What does this objective finding indicate? o 1. Hyperoxemia o 2. Hyperoxia o 3. Hypokalemia o 4. Hypoxemia

o Answer: 4 Individuals whose blood is deficient in oxygen tend to have a bluish discoloration of their skin called cyanosis. Cyanosis is most noticeable in mucous membranes and nailbeds. o Explanation: 1. Hyperoxemia refers to an increased oxygen concentration in the blood measured by arterial oxygen partial pressure (Pa02) values. A Pa02 greater than 120 mm Hg constitutes hyperoxemia. 2. Hyperoxia refers to increased oxygen in tissues and organs. 3. Hypokalemia refers to severe potassium depletion in the circulating blood. The condition is commonly manifested b episodes of muscular weakness or paralysis and postural hypotension. 4. Hypoxemia refers to a decreased oxygen concentration in the blood measured by arterial oxygen partial pressure (Pa02) values. A Pa02 less than 80 mm Ha constitutes hypoxemia.

• A physical therapist assigns a grade of good after performing a manual muscle test with a patient in prone. What is the MOST likely muscle group associated with the described testing procedure? o 1. Hip abductors o 2. Hip adductors o 3. Hip internal rotators o 4. Hip extensors

o Answer: 4 Manual muscle testing is performed in a variety of positions including supine, prone, sidelying, and sitting. The most appropriate manual muscle testing position for a given muscle is based on the muscle's unique orientation and the anticipated muscle grade. A grade of good indicates that the subject completes range of motion against gravity with moderate resistance. o Explanation: 1. The hip abductors would typically be assessed with the patient in sidelying. Muscles that abduct the hip include the gluteus medius, gluteus minimus, piriformis, obturator interns, and tensor fasciae latae. 2. The hip adductors would typically be assessed with the patient in sidelying. Muscles that adduct the hip include the adductor magnus, adductor longus, adductor brevis, and Gracilis. 3. The hip internal rotators would typically be assessed with the patient in short sitting. Muscles that internally rotate the hip include the tensor fasciae latae, gluteus medius, gluteus minimus, pectineus, and adductor longus. 4. The hip extensors are typically tested with the patient in a prone position. Muscles that extend the hip include the gluteus maximus, gluteus medius, semitendinosus, semimembranosus, and biceps femoris.

• A child is seen in physical therapy after being diagnosed with a right thoracic scoliotic curve. Which of the following clinical features would be MOST likely given the patient's diagnosis? o 1. Increased space between the right arm and trunk o 2. Left rib hump with the forward bend test o 3. Excessive anterior tilt of the pelvis o 4. Increased elevation of the right shoulder

o Answer: 4 Scoliosis refers to a lateral curvature of the spine. A patient with a right thoracic scoliotic curve would have the convexity of their curve facing to the right. Signs and symptoms include shoulder level asymmetry with or without the presence of a rib hump. Pain is not typically associated with the spinal curvature, rather it is a result of the abnormal forces placed on other tissues of the body due to the curvature. o Explanation: 1. With a right thoracic scoliotic curve, the convexity of the curve faces to the right and the concavity faces to the left. This positioning of the spine tends to increase the space between the left side of the trunk and the left arm. 2. With a right thoracic scoliotic curve, the convexity of the curve faces to the right. As the disease progresses, the vertebrae also begin to rotate, with the spinous processes moving towards the side of the concavity (i.e., left side). The ribs follow the abnormal rotation of the vertebrae. which results in the ribs on the right side being pushed more posteriorly and the thoracic space being narrowed on this side. This can be detected as a posterior right rib hump, not a left rib hump, during the forward bend test. 3. Scoliosis is a spinal abnormality that primarily occurs in the frontal plane. An excessive anterior tilt of the pelvis is a I sagittal plane abnormality that would be more likely seen in someone with excessive lordosis of the lumbar spine. 4. With a right thoracic scoliotic curve, the convexity of the curve faces to the right and the concavity faces to the left. This relative side bending of the spine places the right shoulder in a more elevated position in comparison to the left shoulder.

• A child is seen in physical therapy after her mother expressed concern that she had a leg length discrepancy. Examination in standing reveals right shoulder elevation compared to the left, unequal spacing between the upper extremities and the trunk, and the head positioned away from the midline. Which condition is MOST consistent with the described clinical presentation? o 1. Spondylolysis o 2. Kyphosis o 3. Spondylolisthesis o 4. Scoliosis

o Answer: 4 Scoliosis refers to a lateral curvature of the spine. Signs and symptoms include shoulder level asymmetry with or without the presence of a rib hump. Pain is not typically associated with the spinal curvature, rather it is a result of the abnormal forces placed on other tissues of the bod due to the curvature. The development of scoliosis is typically idiopathic and is most commonly diagnosed between 10 and 13 ears of age. o Explanation: 1. Spondylolysis refers to a defect in the pars interarticularis or the arch of the vertebra. This is most common in the L5 vertebra but can also occur in other lumbar or thoracic vertebrae. The iniurv most often occurs in children and adolescents who participate in sports that involve repeated stress on the lower back, such as gymnastics, football, and weight lifting. Postural deformities are not typically noted with spondylolysis, although back pain is often present. 2. Kyphosis refers to an excessive curvature of the spine in a posterior direction typically identified in the thoracic spine. The most common symptoms for patients with an abnormal kyphosis include poor posture with a hump appearance of the back, back pain, muscle fatigue, and stiffness 3. In spondylolisthesis, the fractured pars interarticularis separates, allowing the inured vertebra to shift or slip forward on the vertebra directly below it. In children and adolescents, this slippage most often occurs during periods of rapid growth such as an adolescent growth spurt. Back pain is the primary symptom of spondylolisthesis. If the slippage is severe, neurological symptoms may be present. 4. The described clinical presentation is consistent with a scoliotic curvature. The focus of treatment for scoliosis is determined based on the magnitude of the curve and the degree of progression. A patient with a structural curve will present with asymmetries of the shoulders, scapulae. pelvis, and skinfolds. A device called a scoliometer can be used to measure the angle of trunk rotation. The Cobb method is often used to determine the angle of curvature. Patients with scoliosis should be closely monitored for progression of the curve

• A physical therapist treats a patient who has incontinence and reports that her difficulty began after the birth of her son. After completing an examination, the therapist determines that the patient has extremely weak pelvic floor muscles. When instructing the patient in a pelvic floor muscle strengthening program, which of the following positions is the MOST appropriate to initiate treatment? o 1. Sidelying o 2. Sitting o 3. Standing o 4. Supine

o Answer: 4 • A patient with weak pelvic floor muscles should be instructed in strengthening exercises in a position that minimizes the influence of gravity on the pelvic floor. As the patient's strength improves more challenging positions will be utilized for strengthening o Rationale: 1. Although sidelying would be an acceptable position, it is more awkward than the supine position for most patients when initiating a pelvic floor muscle strengthening program. 2. Sitting requires the pelvic floor to contract against the resistance of gravity which would be too challenging for the initiation of pelvic floor exercises. 3. Standing requires the pelvic floor to contract against the resistance of gravity which would be too challenging for the initiation of pelvic floor exercises. 4. Patients with extremely weak pelvic floor muscles should initiate strengthening exercises in a horizontal plane in order to avoid gravity exerting a downward force on the pelvic floor

• A patient with atrial flutter suddenly becomes extremely lightheaded while exercising and appears to lose consciousness. Which of the following physiologic responses BEST explains the change in the patient's status? o 1. Increased filling of the ventricles o 2. Increased system congestion o 3. Decreased atrial depolarization o 4. Decreased blood traveling to the brain

o Answer: 4 • Atrial flutter is a common abnormal heart rhythm characterized by rapid atrial tachycardia. This rapid rate creates decreased filling time of the ventricles resulting in diminished amounts of blood being ejected from the heart. This form of arrhythmia produces sawtooth shaped P waves. o Rationale: 1. The rapid rate of atrial contraction (i.e., 250-350 beats per minute) associated with atrial flutter results in decreased filling time of the ventricles and diminished amounts of blood being ejected from the heart. 2. Increased system congestion is more characteristic of congestive heart failure than a supraventricular arrhythmia such as atrial flutter. Congestive heart failure is often caused by diminished pumping ability of the ventricles due to muscle weakening (systolic dysfunction) or to stiffening of the heart muscle that impairs the ventricles' capacity to relax and fill (diastolic dysfunction). 3. Atrial flutter is characterized by an extremely rapid rate of atrial depolarization (i.e.. 250-350 beats per minute). This rapid rate creates decreased filling time of the ventricles resulting in diminished amounts of blood being ejected from the heart. 4. Atrial flutter results in decreased blood traveling to the brain as well as other areas of the body. As a result vital organs such as the heart muscles and brain may not receive enough blood causing mild symptoms (e.g. palpitations, lightheadedness, weakness) or more serious symptoms (e.g., fainting, angina, organ failure).

• A physical therapist initiates an exercise program for a patient rehabilitating from cardiac surgery. During the treatment session, the therapist monitors the patient's oxygen saturation rate. Which of the following would be MOST representative of a normal oxygen saturation rate? o 1. 82% o 2. 87% o 3. 92% o 4. 97%

o Answer: 4 • Oxygen saturation (Sa02) measures the percentage of hemoglobin saturated with oxygen. The normal range for oxygen saturation is between 95-98% in health individuals. o Rationale: 1. A value of 82% Sa02 demonstrates significant hypoxemia resulting in the patient requiring continuous use of supplemental oxygen. Exercise would be contraindicated at this level 2. A value of 87% Sa02 is below the range for acceptable oxygen saturation. The patient would likely use supplemental oxygen at rest and with exercise. The patient should maintain 90% Sa02 or better with supplemental oxygen use 3. A value of 92% Sa02 is not within normal limits for oxygen saturation. In most cases, the patient would be monitored to ensure that Sa02 does not fall below 90% during exertion or exercise 4. A value of 97% Sa02 is within the specified range of 95-98% for normal arterial oxygen saturation.

• A physical therapist works on therapeutic positioning with a patient who has chronic pulmonary disease. The patient has significant weakness of the diaphragm and is hypertensive. Which of the following positions is the MOST appropriate to initiate treatment? o 1. Prone o 2. Supine o 3. Trendelenburg o 4. Reverse Trendelenburg

o Answer: 4 • The reverse Trendelenburg position refers to a position in which the patient's head is elevated on an inclined plane in relation to the feet o Rationale: 1. The prone position would be a difficult position in which to teach the patient diaphragmatic breathing since the weight of the abdominal contents on the diaphragm makes it more difficult for a weakened diaphragm to contract. 2. In supine, the weight of the abdominal contents on the diaphragm makes it more difficult for a weakened diaphragm to contract. The supine position can also reduce the functional residual capacity of the lungs by as much as fifty percent. 3. In the Trendelenburg position the patient's head is lower than their feet. The position is used to facilitate drainage from the lower lobes of the lungs and to increase blood pressure in hypotensive patients. The position would tend to increase the blood pressure of a patient that is already hypertensive 4. The reverse Trendelenburg position is recommended to reduce hypertension and facilitate movement of the diaphragm by using gravity to reduce the weight of the abdominal contents on the diaphragm.

A patient complains of an insidious onset of heel pain that is most painful when initially weight-bearing after periods of inactivity. The patient demonstrates decreased dorsiflexion and tenderness over the medial calcaneal tubercle. What is the patient's MOST likely diagnosis? • 1. Plantar fasciitis • 2. Achilles tendinitis • 3. Tarsal tunnel syndrome • 4. Calcaneal stress fracture

• Answer: 1 • Rationale: 1. Plantar fasciitis typically presents insidiously with pain originating in the heel. Symptoms are usually exaggerated with initial weight bearing after a period of rest and decrease with activity. Decreased ankle dorsiflexion, pain with palpation over the medial calcaneal tubercle, and an antalgic gait are common examination findings. 2. Achilles tendinitis may present insidiously or in response to repetitive strain. Tenderness to palpation, edema or a nodular formation may be noted proximal to the calcaneal insertion of the tendon. Decreased plantarflexion strength, pain with Achilles tendon stretching, and decreased dorsiflexion range of motion are common examination findings. 3. Tarsal tunnel syndrome may present due to various etiologies including trauma and altered bony alignment. Symptoms typically include intermittent burning or tingling sensations along the medial foot and ankle which may progress to muscle weakness. Pain at the end range of dorsiflexion and a positive Tinel's sign over the posterior tibial nerves are common examination findings 4. Calcaneal stress fractures typically develop as a result of cumulative trauma. Complaints of pain during inactivity or at night assist the therapist to differentiate the condition from plantar fasciitis. Examination findings commonly include pain when squeezing the heel from both sides. Pain is typically exacerbated by activity and persistent during periods of non-weight bearing inactivity.

A physical therapist measures body composition using skinfold measurements prior to initiating an exercise program. When measuring the abdominal skinfold, what is the MOST appropriate method? • 1. Utilize a vertical fold approximately 2 cm to the right of the umbilicus • 2. Utilize a horizontal fold approximately 2 cm to the right of the umbilicus • 3. Utilize a vertical fold approximately 2 cm to the left of the umbilicus • 4. Utilize a horizontal fold approximately 2 cm to the left of the umbilicus

• Answer: 1 • Rationale: 1. The abdominal skinfold site utilizes a vertical fold approximately 2 cm to the right of the umbilicus. 2. The abdominal skinfold requires the use of a vertical fold and not a horizontal fold. 3. All skinfold measurements are made using the right side of the body. 4. The abdominal skinfold requires the use of a vertical fold using the right side of the body

A physical therapist prepares to apply a sterile dressing to a wound after debridement. The therapist begins the process by drying the wound using a towel. The therapist applies medication to the wound using a gauze pad and then applies a series of dressings that are secured using a bandage. The application of which step would NOT warrant the use of sterile technique? • 1. Bandage • 2. Dressings • 3. Medication • 4. Towel

• Answer: 1 • Rationale: 1. A bandage is applied over a dressing. The function of a bandage is to keep the dressing in position, provide a barrier between the dressing and the environment, provide pressure, and protect the wound. Since the bandage does not come in direct contact with the area surrounding the wound, sterile technique is not required. 2. A dressing for a wound is usually comprised of several layers. The function of a dressing is to prevent contamination to the wound, prevent microorganisms within the wound from infecting other areas, assist with healing, apply pressure, absorb drainage, and prevent further injury to the wound. Application of all layers of a dressing requires sterile technique. 3. The application of medication is part of the dressing in this scenario and should be applied using sterile technique 4. If the patient is using the towel directly on the area of the wound. the towel must be sterile and the therapist must use sterile technique to avoid contamination.

A physical therapist educates a patient status post transfemoral amputation on the importance of frequent skin checks. What is the MOST appropriate resource for the patient to utilize when inspecting the posterior aspect of the residual limb? • 1. Hand mirror • 2. Video camera • 3. Caregiver • 4. Prosthetist

• Answer: 1 • Rationale: 1. A patient should regularly inspect all areas of the residual limb using a mirror in order to maintain healthy skin. Proper skin care is important for all patients following amputation. 2. A video camera would not be the most appropriate resource to inspect the residual limb since it can be cumbersome to use and may not be readily available for all patients. 3. A caregiver is capable of viewing areas of the residual limb that the patient cannot see, however, this option would result in the patient being dependent on another person. 4. A prosthetist, like a caregiver, could inspect the skin, however, this option would result in the patient being dependent on another person.

A patient that required a mechanical ventilator for two weeks following a near-drowning incident is cleared to gradually decrease use of the device. Which measured cardiopulmonary value would indicate a sign of distress during the weaning process? • 1. Respiratory rate of 38 breaths per minute • 2. Tidal volume of 350 milliliters • 3. Pulse oximetry measured at 91 percent • 4. Heart rate change of 10 beats per minute over baseline

• Answer: 1 • Rationale: 1. A significantly elevated respiratory rate (e.g., 38 breaths per minute) is a cardinal sign of distress during the weaning process. Persistent tachypnea may result in failure to wean from the mechanical ventilator. 2. A tidal volume of 350 milliliters would not be considered a sign of distress during the weaning process. A tidal volume value less than 325 milliliters would be considered a sign of distress. 3. A pulse oximetry value of 91 percent would not be considered a sign of distress during the weaning process. A pulse oximetry value less than 90 percent would be considered a sign of distress 4. A heart rate change of 10 beats per minute over baseline would not be considered a sign of distress during the weaning process. A heart rate change of greater than 20 beats per minute would be considered a sign of distress.

A physical therapist observes that a patient with chronic venous insufficiency is wearing stockings with 20 mm Hg of pressure despite being prescribed stockings with 40 mm Hg of pressure. Which clinical finding would be MOST anticipated? • 1. Edema • 2. Loss of sensation in the feet • 3. Angina pectoris • 4. Hyporeflexia

• Answer: 1 • Rationale: 1. An individual who has decreased venous return and is wearing stockings with insufficient pressure is likely to experience symptoms of chronic venous insufficiency, including edema. 2. While loss of sensation in the feet is possible if a patient is wearing a garment that is too tight, loss of sensation in the feet is not commonly observed in patients with chronic venous insufficiency. This finding would be more likely observed in patients with diabetes. 3. Angina pectoris is a symptom of myocardial infarction and is not a symptom of chronic venous insufficiency. 4. Hyporeflexia refers to a diminished or absent response to tapping of the tendon. This can be indicative of disease that involves one or multiple components of the reflex arc itself, such as a lower motor neuron disorder. Reflexes would not be significantly impacted based on the change in stocking pressure. An individual who has decreased venous return and is wearing stockings with insufficient pressure is likely to experience symptoms of chronic venous insufficiency, including edema.

A patient diagnosed with epilepsy has a history of generalized tonic-clonic seizures. Which class of pharmacological agent would MOST likely be prescribed to treat this type of seizure? • 1. Barbiturates • 2. Anticholinergics • 3. Cholinergic stimulants • 4. Beta-adrenergic agonists

• Answer: 1 • Rationale: 1. Barbiturates are a class of drugs often prescribed in the treatment of seizures, but especially in the treatment of generalized tonic-clonic seizures and both simple and complex partial seizures. Barbiturates function to decrease neuronal activity in the brain by increasing the inhibitory effects of GABA 2. Anticholinergics are a class of drugs used to treat certain gastrointestinal disorders, such as peptic ulcer disease or irritable bowel syndrome. Other uses for these medications include the treatment of Parkinson's disease, cardiac arrhythmias, motion sickness, urinary frequency and incontinence, and bronchoconstriction. 3. Cholinergic stimulants are a class of drugs used to treat the decrease in smooth muscle tone in the gastrointestinal tract or urinary bladder that may occur following abdominal surgery or trauma. Other uses for these medications include the treatment of Alzheimer's disease, glaucoma, and myasthenia gravis. 4. Beta-adrenergic agonists are a class of drugs used to treat conditions that involve bronchoconstriction. These medications cause smooth muscle relaxation within the bronchioles resulting in bronchodilation.

A patient two days following Cesarean delivery complains of incisional pain with coughing and sneezing. What is the MOST appropriate initial physical therapy intervention? • 1. Brace the incision with a pillow when coughing or sneezing • 2. Contract the pelvic floor muscles • 3. Perform friction massage around the incision • 4. Initiate postural awareness activities

• Answer: 1 • Rationale: 1. Bracing an abdominal incision with a pillow during a cough or sneeze supports healing tissues by limiting the abdominal pressure associated with the activity. Instructing the patient in this technique provides an appropriate and immediate method of improving overall comfort. 2. Contraction of the pelvic floor muscles may be helpful to limit the pain associated with postpartum pelvic floor dysfunction, but it is unlikely to relieve the abdominal incision pain associated with coughing or sneezing. 3. Friction massage may assist in limiting future pain by minimizing adhesion formation. However, it will not specifically influence the patient's immediate complaints. Patients may begin friction massage once the incision is sufficiently healed. 4. Pregnancy-related faulty posture is frequently the result of muscle imbalance in the trunk. Postural correction and strengthening may assist in preventing future pain due to muscle imbalance, but they will not specifically influence the patient's immediate complaints.

A physical therapist is treating a patient with adhesive capsulitis in an outpatient setting. The patient has a history of gastroesophageal reflux disease and is currently treated with Prevacid. Which position would potentially be the MOST problematic for the patient when performing upper extremity exercises? • 1. Recumbent position on a mat table in right sidelying • 2. Standing on level surfaces • 3. Sitting with hips and knees flexed to 90 degrees • 4. Standing without support on a foam surface

• Answer: 1 • Rationale: 1. Considerations for a patient that presents with GERD would include avoiding certain exercises, avoiding exercising in a recumbent position, avoiding the right sidelying position, and avoiding tight clothing. 2. This patient should be able to stand on level surfaces and perform upper extremity exercises without increasing symptoms of GERD. There is no indication that the patient has other co-morbidities that would limit their ability to exercise in this position. 3. The patient should be able to perform an upper extremity exercise program in sitting without increasing symptoms of GERD. There is no indication that the patient has other co-morbidities that would limit their ability to exercise in this position. 4. Standing without support on a foam surface may be difficult for someone with balance deficits, especially within the somatosensory system. Standing on a foam surface will not impact or increase symptoms of GERD.

A patient being treated for shoulder impingement complains of muscle soreness in the triceps muscles for two days following a physical therapy session. Which of the following exercises would MOST likely have resulted in the stated complaint? • 1. Eccentric bench press with dumbbells • 2. Concentric bench press with dumbbells • 3. Concentric pectoral fly with resistance bands • 4. Eccentric pectoral fly with resistance bands

• Answer: 1 • Rationale: 1. DOMS is most commonly noted in patients who have engaged in high intensity, eccentric strengthening exercises, especially if the patient has recently begun a resistance training program. Since the patient is eccentrically strengthening the triceps muscles in this exercise, they are at the highest risk for developing DOMS. 2. The bench press exercise could potentially cause muscle soreness in the triceps muscles since the triceps help to extend the elbows during this exercise. However, since the focus is on the concentric phase of the exercise, DOMS is less likely to occur. 3. The pectoral fly exercise involves horizontal adduction of both shoulders against resistance. The triceps are not involved with horizontal adduction and would not be activated during this exercise. 4. During the pectoral fly exercise, the same muscles are working during both the concentric and eccentric phases of the exercise. During the eccentric phase, the muscles are producing force while elongating to control the motion back to the starting position.

A physical therapist is considering the use of electrical stimulation on a chronic wound. Which of the following sets of parameters is the MOST appropriate? • 1. Monophasic, direct current • 2. Biphasic, direct current • 3. Monophasic, alternating current • 4. Biphasic, alternating current

• Answer: 1 • Rationale: 1. HVPC utilizes monophasic direct current in order to enhance healing of chronic wounds. Monophasic current is a type of pulsatile current that produces a polarity effect in tissues. Direct current is characterized by a constant flow of electrons from the anode (i.e., positive electrode) to the cathode (i.e., negative electrode). 2. While HVPC utilizes direct current in order to enhance the healing of chronic wounds, it utilizes monophasic pulsed current rather than biphasic pulsed current. Biphasic pulsed current has two phases, one which is positive and one which is negative. An example of electrical stimulation utilizing biphasic current is interferential current. 3. While HVPC utilizes monophasic current, it utilizes direct current rather than alternating current. Alternating current is characterized by polarity that continuously changes from positive to negative with the change in direction of current flow. Alternating current is biphasic, symmetrical, or asymmetrical and is characterized by a waveform that is sinusoidal in shape. 4. HVPC utilizes monophasic direct current rather than biphasic alternating current. An example of electrical stimulation that utilizes biphasic alternating current is neuromuscular electrical stimulation.

A physical therapist examines a patient post surgery. The patient has diabetes, however, has no other significant past medical history. Which of the following situations would MOST warrant immediate medical attention? • 1. Signs of confusion and lethargy • 2. Systolic blood pressure increase of 20 mm Hg during exercise • 3. Lack of significant clinical findings following the examination • 4. Discovery of significant past medical history unknown to the physician

• Answer: 1 • Rationale: 1. It is important to treat hypoglycemia immediately using glucose tablets or sugar in order to raise blood glucose levels. 2. An increase in systolic blood pressure of 20 mm Hg is a normal response to exercise. 3. The lack of significant findings in the examination does not warrant immediate medical attention 4. Discovery of significant unknown medical history warrants referral to the physician for possible future examination and follow-up, but does not warrant immediate medical attention.

An elderly patient who has moderate osteoarthritis of the knees has been referred to physical therapy for an exercise program. Which of the following would be the MOST appropriate activity to help maintain healthy articular cartilage? • 1. Walking program • 2. Plyometric exercises • 3. Jogging on a treadmill • 4. Lunges

• Answer: 1 • Rationale: 1. Moderate pain free exercise on a regular basis is typically the most desirable activity for a patient with osteoarthritis. Nourishment of articular cartilage is facilitated by the "milking" action of articular surface deformation during intermittent loading. A walking program at a comfortable pace and distance is an example of an activity meeting this criteria. 2. Plyometric exercise uses explosive movements to develop muscular power. This type of exercise would be too provocative for an elderly patient with osteoarthritis. 3. Jogging on a treadmill would likely exacerbate the patient's current condition. Symptoms commonly associated with osteoarthritis include pain, stiffness, and effusion. 4. Lunges are a dynamic training exercise used primarily to strengthen the quadriceps and gluteal muscles. This form of exercise can be effective, however, would typically be too aggressive for an elderly patient with osteoarthritis.

A physical therapist completes documentation after administering an ultrasound treatment. Which of the following treatment parameters would be the LEAST important to document? • 1. Patient position • 2. Treatment time • 3. Intensity • 4. Duty cycle

• Answer: 1 • Rationale: 1. Patient position is primarily determined based on accessibility to the body surface being treated and patient comfort. Although this is an important item to consider when using ultrasound, it is not as critical as the specific parameters of the ultrasound treatment. 2. The treatment time refers to the period of time that ultrasound is being emitted. The treatment time when using ultrasound is primarily determined based on the size of the surface area to be treated. Given the variability in treatment times when using ultrasound, it is a necessary parameter to document. 3. Intensity is a measure of the rate at which energy is being delivered per unit of area. Intensity levels vary considerably based on the desired physiologic effects and therefore, it is a necessary parameter to document. 4. Duty cycle is defined as the ratio of the on-time to the total time. The duty cycle selected will significantly influence changes in tissue temperature and therefore, it is a necessary parameter to document.

A physical therapist monitors the lab values of an adult male patient ambulating in an acute care hospital following coronary artery bypass graft surgery. The medical chart notes that the patient's hematocrit value is 68%. Which condition would MOST likely have led to this lab value abnormality? • 1. Polycythemia vera • 2. Sickle cell anemia • 3. Colon cancer • 4. Gastric ulcers

• Answer: 1 • Rationale: 1. Polycythemia vera is a blood cancer in which the bone marrow makes too many red blood cells. The increase in red blood cell production would lead to an elevated hematocrit value. Other conditions that may lead to an elevated hematocrit include dehydration, pulmonary fibrosis, and cor pulmonale. 2. Sickle cell anemia is a condition in which red blood cells are misshapen, which leads to inadequate oxygenation of the body's tissues. Because this condition results in fewer healthy red blood cells, it would more likely lead to a low hematocrit value. 3. Cancers can lead to a decrease in the production of red blood cells, which results in a low hematocrit value. Colon cancer specifically leads to bleeding within the gastrointestinal tract. Blood loss is another cause of a low hematocrit value. 4. Gastric ulcers, sores that form in the lining of the stomach, lead to bleeding within the body. This blood loss lowers the numbers of red blood cells, leading to a low hematocrit value.

A physical therapist works on transfer training with a patient who is currently taking a diuretic medication. Which side effect is MOST likely to occur during the session secondary to the use of this type of medication? • 1. Postural hypotension • 2. Bleeding • 3. Sedation • 4. Headache

• Answer: 1 • Rationale: 1. Postural hypotension (lightheadedness or dizziness associated with a change in position) is a common side effect associated with the use of diuretics due to a reduction in blood volume. Postural hypotension could occur during transfer training since the activity requires frequent position changes. 2. Though there is a risk for bleeding to occur while performing transfers, bleeding is not a side effect associated with the use of diuretics. Bleeding is a side effect more commonly associated with thrombolytic medications. 3. Though sedation may affect a patient's ability to perform transfers, it is not a side effect associated with the use of diuretics. Sedation is a side effect more commonly associated with narcotic or antidepressant medications 4. Headache is a side effect that is associated with the use of some diuretic medications. However, this side effect is less common than postural hypotension and would not necessarily be associated with transfer training.

A physical therapist treats a patient diagnosed with lateral epicondylitis. The patient exhibits pain which limits resisted testing and tenderness with soft tissue palpation that is consistent with the diagnosis. Which muscle is MOST likely to exhibit the described findings? • 1. Extensor carpi radialis brevis • 2. Flexor carpi radialis • 3. Flexor carpi ulnaris • 4. Brachioradialis

• Answer: 1 • Rationale: 1. The ECRB originates from the lateral epicondyle of the humerus and inserts dorsally at the base of the third metacarpal. The muscle's actions include wrist extension and radial deviation. 2. The flexor carpi radialis originates from the medial epicondyle of the humerus and inserts at the bases of the second and third metacarpals. The muscle's actions include wrist flexion and radial deviation. 3. The flexor carpi ulnaris originates from the medial epicondyle of the humerus and inserts at the pisiform,hook of the hamate, and base of the fifth metacarpal. The muscle's actions include wrist flexion and ulnar deviation. 4. The brachioradialis originates from the lateral supracondylar ridge of the humerus and inserts at the styloid process of the radius. The muscle's action includes elbow flexion.

A physical therapist works with a nine-year-old child with cystic fibrosis. What age BEST approximates the median age of survival for a patient with cystic fibrosis? • 1. 15 years old • 2. 25 years old • 3. 35 years old • 4. 50 years old

• Answer: 3 • Rationale: 1. The median age of survival for a patient with cystic fibrosis is well beyond 15 years of age. More than 70% of patients with cystic fibrosis are diagnosed by two years of age 2. The median age of survival for a patient with cystic fibrosis is beyond 25 years of age. More than 45% of patients with cystic fibrosis are age 18 years or older. 3. The median age of survival for a patient with cystic fibrosis is currently in the mid-thirties. This age of survival has steadily increased each decade due to advances in treatment which has resulted in many individuals with cystic fibrosis living well into adulthood. 4. Although some patients with cystic fibrosis can live beyond 45 years of age, the median age of survival remains less than 40 years.

A patient sustains a deep laceration on the anterior surface of the forearm. The physical therapist attempts to stop the bleeding by applying direct pressure over the wound, but is unsuccessful. The MOST appropriate action is for the therapist to attempt to apply compression over the pressure point of which of the following arteries? • 1. Brachial • 2. Femoral • 3. Radial • 4. Ulnar

• Answer: 1 • Rationale: 1. The brachial artery can be compressed against the medial aspect of the humerus in an attempt to control the bleeding. The pressure point is located on the inside of the arm in the groove between the triceps and biceps, approximately midway between the axilla and the elbow. The brachial artery's location, proximal to the forearm, makes it possible to control bleeding. 2. The femoral artery can be compressed against the pelvic bone. The pressure point is on the front of the thigh immediately below the middle of the crease of the groin where the artery crosses over the pelvic bone as it moves to the leg. Applying pressure to an artery in the lower extremity would not be helpful to control bleeding in the forearm. 3. The radial artery is covered by only fascia and skin at the distal end of the radius and is therefore extremely accessible. The artery is located at the wrist on the volar surface, medial to the styloid process of the radius. The distal location of the artery would not effectively stop bleeding caused by a laceration in the forearm. 4. The radial and ulnar arteries are the two main arteries of the forearm. The ulnar artery lies lateral to the tendon of the flexor carpi ulnaris. Applying pressure to the ulnar artery would not be an effective technique to stop the bleeding.

A physical therapist treats a moderately obese patient in an acute care hospital. The chart review revealed atherosclerosis, diabetes mellitus, and hypertension. The patient presents with an ankle-brachial index value of 0.55. The patient complains of pain with walking that is relieved with rest. Which of the following medical conditions would be MOST consistent with the patient's clinical presentation? • 1. Peripheral vascular disease • 2. Coronary artery disease • 3. Congestive heart failure • 4. Cor pulmonale

• Answer: 1 • Rationale: 1. The clinical presentation of PVD will differ depending on which blood vessel has been compromised. This patient's ankle-brachial index (ABI) indicates moderate peripheral vascular disease and claudication which can result in difficulty performing functional activities. 2. Coronary artery disease refers to the narrowing or blockage due to fatty build-up (cholesterol) within the artery walls reducing the overall blood flow to the cardiac muscle. Symptoms will vary based on the location and severity of blockage. These symptoms can include nausea, vomiting, heartburn, shortness of breath, chest pain, and profuse sweating. 3. Congestive heart failure (CHF) is a progressive condition in which the heart cannot maintain a normal cardiac output to meet the body's demands for blood and oxygen. The ventricles weaken and dilate to the point that the heart can't pump efficiently. The etiology of congestive heart failure may include arrhythmia, pulmonary embolism, hypertension, valvular heart disease, myocarditis, unstable angina, renal failure, and severe anemia. A patient with CHF will initially show signs of tachycardia. As the severity of CHF increases, signs of venous congestion usually become apparent. 4. Cor pulmonale, also known as pulmonary heart disease, refers to hypertrophy of the right ventricle caused by altered structure or function of the lung. The cardinal symptom is progressive shortness of breath, especially with exertion. Other symptoms include fatigue, atypical chest pain, swelling of the lower extremities, dizziness, and syncope.

A patient with cardiac arrhythmias has been prescribed beta-blockers. What is the MOST effective way to monitor the patient's activity tolerance? • 1. Rating of perceived exertion • 2. Visual analog scale • 3. Palpation of radial artery • 4.Assessment of telemetry strip

• Answer: 1 • Rationale: 1. The rating of perceived exertion scale, or "Borg's scale, is a method of subjectively assessing exercise intensity. It is the preferred method for exercise prescription for a patient taking beta-blockers. 2. The visual analog scale is a pain scale and would not be appropriate to measure activity tolerance 3. Beta-blockers mute the heart rate response to exercise by decreasing sympathetic input to the myocardium. As a result, the palpation of heart rate is not an accurate indicator of exercise intensity for patients on beta-blockers. 4. Telemetry is an electrocardiogram monitoring system that provides real-time electrocardiogram results through the radiofrequency transmission of the ECG signal to a monitor. An electrocardiogram is used to study the electrical activity of the heart, including the identification of arrhythmias and heart block. It can also be used to measure heart rate, however, the effectiveness of this measure in assessing activity tolerance would be impacted by the use of beta-blockers.

A seven-month-old infant is able to initiate rolling from supine to prone when playing on a level floor surface. Which scenario would MOST likely be associated with this developmental milestone? • 1. Integration of the tonic labyrinthine reflex • 2. Persistence of the asymmetrical tonic reflex • 3. Integration of the rooting reflex • 4. Persistence of the Galant reflex

• Answer: 1 • Rationale: 1. The tonic labyrinthine reflex is stimulated through the labyrinth in the inner ear. If the infant is in a supine position, the body and extremities are held in extension; in a prone position, the body and extremities are held in flexion. This reflex not only interferes with the ability to roll, but also the ability to prop on elbows, balance in sitting and attain an upright posture from a supine position. Integration of the reflex is often associated with the ability to roll from supine to prone. 2. The asymmetrical tonic neck reflex is stimulated through head position, specifically turning to one side. When the infant turns the head to one side, the arm and leg on the face side are extended while the arm and leg on the scalp side are flexed. The persistence of this reflex will interfere with rolling as well as visual tracking, midline use of the hands, and feeding. 3. The rooting reflex is stimulated by a touch of the cheek. When the infant is stimulated, the infant will turn the head toward the side of stimulation. Integration of the rooting reflex would not have an effect on the ability to roll. 4. The Galant reflex is stimulated by touching the skin along the spine from the shoulder to the hip. The infant's response is lateral flexion of the trunk towards the side of stimulus. Persistence of this reflex interferes with the development of sitting balance.

A physical therapist works with a patient rehabilitating from a recent hospitalization. Which activity would be MOST likely associated with the development of syncope? • 1. Transferring from a supine to sitting position • 2. Riding a stationary bicycle at 50 revolutions per minute • 3. Performing a leg exercise with ankle weights in supine • 4. Walking at 3 miles per hour on a treadmill

• Answer: 1 • Rationale: 1. Transferring from supine to sitting temporarily reduces blood flow to the brain which significantly increases the probability of syncope. This type of response may be related to medications which lower blood pressure, dehydration, significant bleeding, infection, and prolonged bedrest. 2. Riding a stationary bicycle at 50 revolutions per minute is a relatively predictable activity that tends to require a constant amount of exertion. The consistency of the activity makes it less likely to produce syncope than an activity requiring a positional change. 3. Performing a leg exercise with ankle weights in supine tends to be a fairly static activity. The consistency of the activity makes it less likely to produce syncope than an activity requiring a positional change. 4. Walking at 3 miles per hour on a treadmill is a relatively predictable activity that tends to require a constant amount of exertion. The consistency of the activity makes it less likely to produce syncope than an activity requiring a positional change.

A patient in the hospital following surgery sets the target indicator on an incentive spirometer. Which individual would MOST likely have the highest target value when using the device? • 1. 35-year-old female, 66 inches tall, post trans myocardial revascularization • 2. 45-year-old male, 72 inches tall, post total knee arthroplasty • 3. 55-year-old female, 64 inches tall, post-cardiac valve replacement • 4. 55-year-old male, 70 inches tall, post coronary artery bypass graft

• Answer: 2 • Rationale: 1. A 35-year-old female, 66 inches tall, post transmyocardial revascularization would not have the highest target value for SMI using an incentive spirometer of the presented options, as the individuals in options 2 and 4 are taller (larger lung capacities) and male (larger conducting airways and lung size). Although she is the youngest of the presented options, the height and gender differences would be a stronger correlate to lung capacity than the presented age differences. 2. A 45-year-old male, 72 inches tall, post total knee arthroplasty would have the highest target value for SMI using an incentive spirometer of the presented options due to being male and the tallest. The height difference between options 1 and 2 would have a greater influence on lung capacity than the age difference 3. A 55-year-old female, 64 inches tall, post-cardiac valve replacement would likely have the lowest target value for SMI using an incentive spirometer, as this individual is female, older, and has the shortest stature of the presented options. The surgical procedure is generally of less relevance than the other factors. 4. A 55-year-old male, 70 inches tall, post coronary artery bypass graft would have a lower target value for SMI using an incentive spirometer than option 2 as he is older and is 2 inches shorter in height. Although the surgical procedure is typically a less relevant factor, the coronary artery bypass graft would likely also play a role due to sternal precautions and expected pain with lung expansion.

A high school athlete attends an inservice on flexibility training. What is the minimal flexibility training schedule the physical therapist should recommend? • 1. 1 day per week • 2. 3 days per week • 3. 5 days per week • 4. 7 days per week

• Answer: 2 • Rationale: 1. A frequency of 1 day per week fall below any of the recommendations for endurance, strength or flexibility training. 2. Flexibility training should be performed minimally 2-3 days per week, although greater gains in flexibility may occur with greater frequency. 3. A frequency of 5 days per week is often desirable for stretching activities. However, this frequency would not be reflective of the minimal flexibility training schedule. The frequency selected is influenced by the level of intensity or metabolic equivalents of the exercise and the target heart rate range used during exercise. 4. Flexibility training can be performed 5-7 days per week and should include slow and controlled stretching with a hold of 15-30 seconds per stretch. Each stretch should be performed 2-4 times. Although this frequency of stretching is optimal, the minimum is 2-3 days per week.

A physical therapist examines the wound of a patient with diabetes mellitus. The wound is located on the plantar surface of the foot under the head of the first metatarsal and appears non-infected. When asked to flex the great toe, the therapist observes a tendon gliding in the wound bed. Which grade of the Wagner Ulcer Grade Classification Scale BEST describes this wound? • 1. Grade 1 • 2. Grade 2 • 3. Grade 3 • 4. Grade 4

• Answer: 2 • Rationale: 1. A grade 1 ulcer is a superficial ulcer not involving subcutaneous tissue. The described wound has an exposed tendon, therefore penetrating deeper than the subcutaneous tissue. 2. A grade 2 ulcer is a deep ulcer with penetration through the subcutaneous tissue, potentially exposing bone, tendon, ligament or joint capsule. The described wound is consistent with a grade 2 neuropathic ulcer. 3. A grade 3 ulcer is a deep ulcer with osteitis, abscess or osteomyelitis. While the described wound is a deep ulcer, the wound does not show any signs of infection, therefore a grade 3 ulcer is unlikely. 4. A grade 4 ulcer involves gangrene of the affected digit. The described patient does not have gangrene.

A patient returns to physical therapy after completing medical testing using Doppler ultrasonography. Which medical condition would MOST likely necessitate this type of diagnostic imaging? • 1. Peripheral neuropathy • 2. Intermittent claudication • 3. Cardiac arrhythmias • 4. Bursitis

• Answer: 2 • Rationale: 1. A patient with peripheral neuropathy would not be assessed with Doppler ultrasonography. Nerve conduction velocity testing may be used to assess the extent of nerve damage in a patient with peripheral neuropathy. 2. Intermittent claudication is characterized by painful cramping or aching of the lower extremities during physical activity. The condition is associated with impaired blood flow in the lower extremities and thus decreased perfusion to lower extremity muscles. Doppler ultrasonography can be used in a patient with intermittent claudication to determine the extent of impaired arterial blood flow. 3. A patient with cardiac arrhythmias would not be assessed with Doppler ultrasonography. An electrocardiogram may be used to assess the presence of cardiac arrhythmias. 4. Therapeutic ultrasound is a common modality used to treat inflammatory conditions like bursitis. However, Doppler ultrasonography is an assessment tool that would not be used to diagnose bursitis. Bursitis is typically diagnosed through signs and symptoms as opposed to diagnostic imaging.

A physical therapist measures passive forearm pronation and concludes that the results are within normal limits. Which measurement would be classified as within normal limits? • 1. 60 degrees • 2. 80 degrees • 3. 100 degrees • 4. 120 degrees

• Answer: 2 • Rationale: 1. A value of 60 degrees of forearm pronation would be considered hypomobile. 2. According to the American Academy of Orthopedic Surgeons and the American Medical Association normal forearm pronation is 0-80 degrees. 3. A value of 100 degrees of forearm pronation would be considered hypermobile. 4. A value of 120 degrees of forearm pronation would be considered extremely hypermobile and would be unlikely given the joint structure of the superior and inferior radioulnar joints.

During a gait analysis on a patient rehabilitating from a lower extremity injury, the physical therapist measures the number of steps taken by the patient in a 30 second period. The therapist has measured which of the following gait parameters? • 1. Acceleration • 2. Cadence • 3. Velocity • 4. Speed

• Answer: 2 • Rationale: 1. Acceleration is the rate of change of velocity with respect to time 2. Cadence is defined as the number of steps taken by a person per unit of time. Walking with increased cadence decreases the duration of double support time. A cadence of 110 steps per minute is typical in a male, while 116 steps per minute is typical in a female. 3. Velocity is the rate of linear forward motion of the body which is measured most often in centimeters per second, meters per second, or miles per hour. Walking velocity equals distance walked divided by time. 4. Speed is usually classified as slow, free, or fast. Free speed of gait refers to a person's normal walking speed.

A physical therapist recognizes the signs of an airway obstruction for a patient who is eight months pregnant. The patient is choking and unable to independently clear the airway, though they are responsive. What is the MOST appropriate therapist action to help clear the airway? • 1. Administer abdominal thrusts • 2. Administer chest thrusts • 3. Administer rescue breaths • 4. Administer finger sweeps

• Answer: 2 • Rationale: 1. Administering abdominal thrusts to a patient that is eight months pregnant can potentially cause serious injuries to the fetus 2. Chest thrusts are an appropriate alternative to abdominal thrusts since they eliminate the possibility of harming the fetus. A chest thrust is administered with the therapist positioned behind the patient. The therapist places the thumb side of one fist on the sternum and grasps the fist with the other hand. The therapist should provide five chest thrusts in rapid succession. If the patient's airway remains obstructed, the chest thrusts should be repeated 3. Rescue breaths are performed as a part of cardiopulmonary resuscitation when the patient is unresponsive and not breathing. The patient in the described scenario is responsive and would not require rescue breaths. 4. Finger sweeps refer to using the finger to sweep through a patient's mouth in an attempt to remove a foreign object. Finger sweeps are only used when a foreign object is visible.

A patient is unable to take in an adequate supply of nutrients by mouth. As a result, the patient's physician orders the implementation of tube feeding. What type of tube is MOST commonly used for short-term feeding? • 1. Endobronchial • 2. Nasogastric • 3. Endotracheal • 4. Tracheostomy

• Answer: 2 • Rationale: 1. An endobronchial tube, also called Carlen's catheter, is a flexible catheter for bronchospirometry and for isolation of a portion of the lung to control secretions into the remainder of the tracheobronchial tree during general anesthesia. 2. A nasogastric tube is a plastic tube inserted through a nostril that extends into the stomach. The device is commonly used for liquid feeding, medication administration or to remove gas from the stomach. A gastric tube is inserted directly into the stomach for long-term feeding. 3. An endotracheal tube is an airway catheter inserted in the trachea for endotracheal intubation. 4. A tracheostomy refers to an opening made in the trachea in order to insert a catheter or tube, most often to facilitate breathing.

A physical therapist attempts to improve a patient's lower extremity strength. Which proprioceptive neuromuscular facilitation (PNF) technique would be the MOST appropriate to achieve the therapist's goal? • 1. Contract-relax • 2. Repeated contractions • 3. Rhythmic stabilization • 4. Hold-relax

• Answer: 2 • Rationale: 1. Contract-relax is a technique used to increase range of motion. As the extremity reaches the point of limitation the patient performs a maximal contraction of the antagonistic muscle group. The therapist resists the movement followed by relaxation and passive movement into newly gained range of motion. 2. Repeated contractions are used to initiate movement and sustain a contraction through the range of motion. The therapist provides a quick stretch followed by isometric or isotonic contractions. Providing resistance at the point of weakness can enhance the effectiveness of repeated contractions. 3. Rhythmic stabilization is a technique used to increase range of motion and coordinate isometric contractions. The technique requires isometric contractions of all muscles around a joint against progressive resistance. 4. Hold-relax uses isometric contractions to increase range of motion. The contractions are facilitated for all muscle groups at the limiting point within the range of motion. Relaxation occurs and the extremity moves through the newly acquired range to the next point of limitation.

A physical therapist prepares to conduct a gait assessment on a patient recently referred to physical therapy. Which variable would be MOST influential when attempting to estimate the patient's stride length? • 1. Coordination • 2. Height • 3. Strength • 4. Weight

• Answer: 2 • Rationale: 1. Coordination may influence the fluidity of a patient's gait, however, would be unlikely to significantly influence stride length. 2. A patient's height is the primary determinant when estimating stride length since height is positively correlated with limb length and a longer limb will result in a longer stride. 3. The strength of the patient will not significantly influence stride length unless there is dramatic muscular weakness affecting the lower extremities and trunk. 4. The weight of the patient will not significantly influence stride length, although it may be associated with other variables such as efficiency and endurance

An 82-year-old patient reports fatigue when ambulating to and from their home mailbox. Which of the following factors would MOST likely contribute to the patient's reported difficulty? • 1. Increased diastolic blood pressure • 2. Decreased cardiac output • 3. Decreased cardiac afterload • 4. Increased arterial patency

• Answer: 2 • Rationale: 1. Diastolic blood pressure generally will stay the same or decrease with age, while systolic blood pressure increases due to the vascular rigidity that occurs with aging. An increased diastolic blood pressure would not increase the patient's level of fatigue. 2. With increasing age, cardiac output decreases making older adults less tolerant to exercise secondary to decreased perfusion of the peripheral tissues. Diminished cardiac output is associated with exertional fatigue. 3. Cardiac afterload is the force against which the heart must contract to move blood into the systemic circulation. Afterload tends to increase with age. A decreased cardiac afterload would not increase the patient's level of fatigue. 4. Arterial patency decreases with aging due to plaque deposits and diminished elasticity of the arterial walls. The primary impact is an increase in systolic blood pressure with aging. Older adults are at greater risk for coronary artery disease, stroke, and congestive heart failure due to these age-related cardiovascular changes. An increased arterial patency would not increase the patient's level of fatigue.

A physical therapist reviews the medical record of a patient diagnosed with a lower motor neuron disorder. Which finding would be LEAST likely based on the presented information? • Diminished deep tendon reflexes • Clonus • Hypotonia • Fasciculations

• Answer: 2 • Rationale: 1. Diminished deep tendon reflexes (Hyperflexia) are characteristics of lower motor neuron disorders 2. Clonus is characterized by cyclical, spasmodic alteration of muscular contraction and relaxation in response to a sustained stretch of a spastic muscle. This sign is characteristic of upper motor neuron disorders. 3. Hypotonia (i.e., decreased muscular tone) is characteristic of lower motor neuron disorders. 4. Fasciculations are random, spontaneous twitches in a muscle group which may be visible through the skin. They are characteristic of lower motor neuron disorders.

A physical therapist walks with a patient in an inpatient cardiac rehabilitation unit. The patient describes a sensation of lower extremity cramping and sensory changes after a distance of 50 feet. The pain subsides following a brief rest period. The therapist hypothesizes that the patient's symptoms are consistent with pain from claudication. Which of the following tests would be the MOST appropriate to confirm the hypothesis? • 1. Venous Doppler ultrasonography • 2. Arterial Doppler ultrasonography • 3. Clinical electromyography • 4. Electrocardiogram.

• Answer: 2 • Rationale: 1. Doppler ultrasonography is a non-invasive test that evaluates blood flow in major veins, arteries, and the cerebrovascular system. Venous Doppler ultrasonography is performed to evaluate blood flow in the venous system, usually to identify blood clots. 2. Arterial Doppler ultrasonography provides information about the blood flow in the arteries. Arterial Doppler ultrasonography is the most appropriate diagnostic test to administer since the patient presents with claudication pain consistent with arterial insufficiency. 3. Clinical electromyography (EMG) is used to evaluate neuromuscular disease or trauma. Clinical EMG detects and records action potentials from skeletal muscle fibers. Clinical EMG would not provide information about the arterial system. 4. An electrocardiogram (ECG) studies the electrical conduction system of the heart. An ECG would not provide information about the arterial system

A patient suffers a laceration injury to their forearm while exercising in the therapy gym. The physical therapist is having trouble stopping the flow of blood from the wound. Which of the following actions should be the LAST resort when treating the laceration? • 1. Apply pressure directly over the wound • 2. Apply a tourniquet proximal to the site of injury • 3. Elevate the arm above the level of the heart • 4. Apply pressure to the brachial artery

• Answer: 2 • Rationale: 1. In all cases of bleeding, pressure should be applied directly over the wound to help control the bleeding using the appropriate personal protective equipment. Additional steps may need to be taken if this step alone does not slow the flow of blood. 2. Though using a tourniquet can help control bleeding, it may create tissue damage (i.e., necrosis) that is unnecessary, and therefore it is generally only used in cases of severe bleeding. The present scenario is not life-threatening and does not require the use of a tourniquet. 3. In addition to applying pressure to the wound, the therapist may elect to elevate the patient's arm above the level of the heart. This action would make it more difficult for blood to reach the site of the wound and would therefore slow the flow of blood. 4. When bleeding cannot be well controlled, it may be necessary to apply pressure directly to an artery at a point proximal to the wound. This action would result in less blood reaching the wound and would therefore slow the flow of blood. It is important to monitor the duration of the pressure due to the potential for necrosis of the area below the constriction.

A physical therapist working in an acute care hospital treats a patient with transmission-based precautions. The therapist treats the patient in their room and is able to leave the door open during treatment. The therapist is required to wear a mask only once they are within three feet of the patient. This scenario would be MOST appropriate when treating a patient diagnosed with what condition? • 1. MRSA • 2. Mumps • 3. Impetigo • 4. Measles

• Answer: 2 • Rationale: 1. MRSA (methicillin-resistant Staphylococcus aureus) is a pathogen associated with contact precautions. Contact precautions reduce the risk of transmission of infectious agents through direct or indirect contact. Direct contact involves skin-to-skin transmission; indirect contact involves a contaminated intermediate object. Gloves should be used upon entering the patient's room and a gown may be needed for close contact. Both should be discarded prior to exiting the patient's room. 2. Mumps is a pathogen associated with droplet precautions. Droplet precautions reduce the risk of droplet transmission of infectious agents through contact of the mucous membranes of the mouth and nose, contact with the conjunctivae, and through coughing, sneezing, talking or suctioning. Transmission requires close contact, as the infectious agents do not suspend in the air and travel only three feet or less, therefore a mask should be worn when working within three feet of the patient. 3. Impetigo is a pathogen associated with contact precautions. Contact precautions reduce the risk of transmission of infectious agents through direct or indirect contact. Gloves should be used upon entering the patient's room and a gown may be needed for close contact. 4. Measles is a pathogen associated with airborne precautions. Patients with measles typically require a private room with the door closed and monitored negative air pressure. Transport outside the room is extremely limited in order to avoid spreading the pathogen.

A physician's order in the medical record indicates that a patient has been placed on hold from physical therapy due to a suspected pulmonary embolism. Which diagnostic test would be the MOST valid method to confirm a suspected pulmonary embolism? • 1. Magnetic resonance imaging • 2. Pulmonary angiography • 3. Bronchoscopy • 4. Ultrasonography

• Answer: 2 • Rationale: 1. Magnetic resonance imaging (MRI) may be ordered to rule out the presence of additional thrombi, especially in the lower extremities. MRI is a noninvasive procedure that utilizes magnetic fields and radio waves to produce cross-sectional images of the body. MRI can be used to visualize almost any structure within the body, but is most often used for imaging soft tissue structures, such as muscles, menisci, ligaments, tumors, and internal organs. 2. Pulmonary angiography is the most conclusive and accurate means of identifying a pulmonary embolism since it is a test of how blood flows through the lungs. Computed tomographic pulmonary angiography has been reported in the literature to be the imaging test of choice for patients with suspected acute pulmonary embolism. 3. Bronchoscopy is a diagnostic procedure that utilizes direct visualization of the bronchial tree. A bronchoscope is a fiber optic instrument that transmits an image to an eyepiece or video camera and can identify tumors, bronchitis, foreign bodies, and bleeding. Bronchoscopy would not be used to identify pulmonary embolism. 4. Ultrasonography is a noninvasive procedure that uses sound waves to produce images of structures within the body, especially the internal organs (e.g., liver, kidneys). Doppler ultrasound is a specific type of ultrasound commonly used for the detection of deep vein thrombosis, however, ultrasonography is not used to identify pulmonary embolism.

A physical therapist observes a wound that is noticeably red slightly shiny, and mildly moist. What is the therapist MOST likely observing? • 1. Slough • 2.Granulation tissue • 3. Necrotic tissue • 4. Exposed tendon

• Answer: 2 • Rationale: 1. Slough is characterized as tan or yellow, soft, and loosely attached to the base of the wound. 2. Granulation tissue is characterized as pink or red, shiny, and moist. 3. Necrotic tissue is a generic term used to describe dead tissue (i.e., slough or eschar). Eschar is characterized as black or brown, hard, and firmly attached to the base and edges of the wound. 4. While an exposed tendon may appear shiny or moist, it would not appear red in color.

A physical therapist utilizes a manual assisted cough technique on a patient who has a mid-thoracic spinal cord injury. When completing this technique with the patient in the supine position, which of the following locations is the MOST appropriate for placement of the therapist's hands? • 1. Manubrium • 2. Epigastric area • 3. Xiphoid process • 4. Umbilical region

• Answer: 2 • Rationale: 1. The manubrium is the broad, quadrangular-shaped upper part of the sternum. This region is too high to provide effective pressure support for coughing. 2. The epigastric area is the upper central region of the abdomen, located between the costal margins and the subcostal plane. Applying manual hand pressure inwards and upwards over the epigastric area can assist the patient to cough and promote airway clearance. 3. The xiphoid process is a small cartilaginous extension to the lower part of the sternum that is usually ossified in the adult. Pressure over this region should be avoided. 4. The umbilical region is the area surrounding the umbilicus (i.e., belly button). This region is too low to provide effective pressure support for coughing.

When selecting an assistive device for a patient with a lower extremity injury, which of the following patient characteristics should be the LEAST critical for the physical therapist to consider? • 1. Cognitive ability • 2. Height and weight • 3. Upper and lower extremity strength • 4. Level of coordination

• Answer: 2 • Rationale: 1. The patient must possess the cognitive ability to comprehend the supplied instructions and use the assistive device in a manner consistent with the physical therapist's instructions. 2. The majority of assistive devices are appropriate for patients of varying weight and can be readily adjusted (e.g., raised or lowered) to accommodate for different heights. As a result, these variables would be the least critical when selecting an assistive device. 3. A patient's upper and lower extremity strength are critical to assess when selecting an appropriate assistive device. For example, a patient using a swing-through gait pattern would need significant upper extremity strength, but would be less dependent on lower extremity strength. Conversely, a patient using a single cane would need significant lower extremity strength, but would be less dependent on upper extremity strength. 4. Patients must possess a requisite amount of coordination to use specific assistive devices. For example, a patient with poor coordination would likely be able to use a walker, but would have significant difficulty using bilateral canes.

A physical therapist sets the parameters of an iontophoresis unit in preparation for treatment on a patient diagnosed with lateral epicondylitis. Which of the following treatment parameters would represent the highest iontophoresis dose? • 1. Current amplitude of 1.0 milliamps; duration of 20 minutes • 2. Current amplitude of 2.0 milliamps; duration of 10 minutes • 3. Current amplitude of 3.0 milliamps; duration of 20 minutes • 4. Current amplitude of 4.0 milliamps; duration of 10 minutes

• Answer: 3 • Rationale: 1. A current amplitude of 1.0 milliamps delivered for 20 minutes would result in a dosage of 20 mA-min. 2. A current amplitude of 2.0 milliamps delivered for 10 minutes would result in a dosage of 20 mA-min. 3. A current amplitude of 3.0 milliamps delivered for 20 minutes would result in a dosage of 60 mA-min. 4. A current amplitude of 4.0 milliamps delivered for 10 minutes would result in a dosage of 40 mA-min.

A physical therapist contemplates the necessary amount of force to overcome friction when performing mechanical lumbar traction using a non-split traction table. What percentage of a patient's body weight would be MOST representative of this value? • 1. 5 percent • 2. 15 percent • 3. 25 percent • 4. 50 percent

• Answer: 3 • Rationale: 1. A force of 5 percent of the patient's body weight would be insufficient to overcome the force of friction. This would correspond to 7.5 pounds for a patient weighing 150 pounds. 2. A force of 15 percent of the patient's body weight would be insufficient to overcome the force of friction. This would correspond to 22.5 pounds for a patient weighing 150 pounds. 3. A force of 25 percent of the patient's body weight would be consistent with the amount of force necessary to overcome the force of friction. This would correspond to 37.5 pounds for a patient weighing 150 pounds. This value can be estimated by multiplying the percentage of the body weight below L3 (i.e., 50 percent) and the coefficient of friction (0.5). The coefficient of friction refers to the constant frictional forces when applying traction between surfaces. The coefficient of friction of the human body on a mattress is 0.5. The result is 25 percent, meaning that a force of 25 percent would be necessary to overcome the force of friction. The use of a split traction table can significantly decrease the effects of friction. 4. A force of 50 percent of the patient's body weight would be far more than the amount of force necessary to overcome the force of friction. This value would correspond to 75 pounds for a patient weighing 150 pounds. A force of 50 percent of the patient's body weight would be necessary for true mechanical separation of vertebrae.

A patient reports brief episodes of significant dizziness when standing during the last 24 hours. The patient states that their symptoms began shortly after they began taking Lasix to treat congestive heart failure. What is the MOST likely rationale for the reported dizziness? • 1. Hypertension due to a decrease in blood plasma volume • 2. Hypertension due to an increase in blood plasma volume • 3. Hypotension due to a decrease in blood plasma volume • 4. Hypotension due to an increase in blood plasma volume

• Answer: 3 • Rationale: 1. Diuretics are often utilized to reduce blood pressure and symptoms of congestive heart failure. This occurs as a result of decreased blood plasma volume and cardiac preload. Orthostatic hypotension, rather than hypertension, is a common side effect of diuretic agent use. 2. An increase in blood plasma volume may be associated with a significant increase in blood pressure resulting in a hypertensive state. However, diuretic agents act to reduce both blood plasma volume and blood pressure. 3. Diuretics act to reduce blood plasma volume by enhancing the excretion of water and electrolytes by the kidneys. This action increases the likelihood of subjective complaints associated with orthostatic hypotension since reduced plasma volume makes it difficult for the body to quickly adapt to positional changes. 4. A hypotensive state is consistent with the patient's described complaints and the known potential side effects of diuretic agents. However, diuretics effectively reduce rather than increase blood plasma volume.

A physical therapist discusses the importance of a proper diet with a patient diagnosed with congestive heart failure. Which of the following substances would MOST likely be restricted in the patient's diet? • 1. High-density lipoproteins • 2. Low-density lipoproteins • 3. Sodium • 4. Triglycerides

• Answer: 3 • Rationale: 1. High-density lipoproteins (HDL) are the smallest particles in the classes of lipoproteins. They are composed of proteins, cholesterol, and a small amount of triglyceride HDL plays an important role in lipid metabolism by transporting cholesterol back to the liver from the cells. High levels of HDL reduce the incidence of coronary artery disease. There is no association between HDL and congestive heart failure. 2. Low-density lipoproteins (LDL) are the major carriers of cholesterol in plasma. Elevated LDL is a cause of coronary artery disease. There is no association between LDL and congestive heart failure. 3. Due to poor cardiac output in congestive heart failure, renal and extrarenal sensors initiate a process to retain fluid to increase arterial blood flow. Retention of sodium is part of that process. By controlling sodium intake and water retention, congestive heart failure can be more effectively controlled. 4. Triglycerides are combinations of glycerol and fatty acids. Elevated triglycerides are not independently predictive of coronary artery disease, but are associated with known risk factors for atherosclerosis, including a low HDL cholesterol level and uncontrolled diabetes. There is no association between triglycerides and congestive heart failure.

A patient recently diagnosed with a deep venous thrombophlebitis is placed on heparin. What is the PRIMARY side effect associated with heparin? • 1. Hypotension • 2. Depression • 3. Excessive anticoagulation • 4. Thrombocytopenia

• Answer: 3 • Rationale: 1. Hypotension, a lower than normal systolic or diastolic blood pressure, is not a side effect of heparin. 2. Depression, a mood disorder characterized by loss of interest or pleasure in living, is not a side effect associated with heparin. 3. The most common side effect of heparin is impaired clotting ability. A physical therapist should be careful to avoid excessive contact or bumping of the limbs of a patient on heparin since this may cause bruising or bleeding. 4. Thrombocytopenia, an abnormal decrease in the number of blood platelets, has been associated with heparin use but is not the primary side effect.

A patient uses a self-administered assessment tool as a method to record daily progress. What type of reliability would be the MOST essential using this tool? • 1. Reliability of parallel forms • 2. Internal consistency • 3. Intratester • 4. Intertester

• Answer: 3 • Rationale: 1. Reliability of parallel forms refers to the consistency between results of two tests constructed in the same way from the same content domain. Parallel forms reliability is not an issue in this example because there is only one form of the assessment tool. 2. Internal consistency reliability refers to the consistency of results across items within a test. The reliability of the instrument is evaluated by estimating how well the different items within the test, which are supposed to reflect the same construct, do yield similar results. This is not of concern in the example. 3. Intratester reliability refers to the extent to which scores on the tool obtained by the same tester are consistent. The tool should have intratester reliability so that any changes recorded can be attributed to progress in therapy and not to unreliable measurement. 4. Intertester reliability refers to the extent of agreement of the scores recorded by two or more individuals. Interrater reliability addresses the consistency of the implementation of a rating system.

A patient status post CVA exhibits significant weakness in the affected lower extremity. When strengthening the affected abductor muscle group using irradiation, which muscle group on the unaffected lower extremity would the physical therapist utilize to resist a maximal isometric contraction? • 1. Hip flexor muscles • 2. Hip extensor muscles • 3. Hip abductor muscles • 4. Hip lateral rotator muscles

• Answer: 3 • Rationale: 1. Resisting the unaffected hip flexor muscles will typically increase muscle tension in the contralateral hip flexors. 2. Resisting the unaffected hip extensor muscles will typically increase muscle tension in the contralateral hip extensors 3. Resisting the unaffected hip abductor muscles will typically increase muscle tension in the contralateral hip abductors. 4. Resisting the unaffected hip lateral rotator muscles will typically increase muscle tension in the contralateral hip lateral rotators.

A physical therapist completes a posture screening and muscle length test of the hip flexors on a patient. The therapist determines that the patient has extremely tight hip flexors bilaterally. What common structural deformity is MOST often associated with tight hip flexors? • 1. Scoliosis • 2. Exaggerated kyphosis • 3. Exaggerated lordosis • 4. Spondylolysis

• Answer: 3 • Rationale: 1. Scoliosis refers to a lateral curvature of the soine. Scoliosis can occur in the cervical, thoracic or lumbar spine. Classifications of scoliosis include idiopathic, non-structural, and structural. Scoliosis is not necessarily associated with tight hip flexors. 2. Kyphosis refers to excessive curvature of the spine in a posterior direction usually identified in the thoracic spine. A structural change in the thoracic spine would not necessarily be associated with tight hip flexors. 3. Lordosis refers to an excessive curvature of the spine in an anterior direction, usually identified in the cervical or lumbar spine. Tight hip flexors are often associated with excessive lordosis (anterior pelvic tilt) due to the origin and insertion of the hip flexors. 4. Spondylolysis refers to a defect in the pars interarticularis or the arch of the vertebra. This is most common in the L5 vertebra, but can also occur in other lumbar or thoracic vertebra.

A patient begins to cry in the middle of a treatment session The physical therapist attempts to comfort the patient, however, eventually has to discontinue treatment. What section of the S.O.A.P. note would be the MOST appropriate to document the incident? • 1. Subjective • 2. Objective • 3. Assessment • 4. Plan

• Answer: 3 • Rationale: 1. The subjective section refers to information the patient communicates directly to the physical therapist. This could include patient statements, social history, medical history or patient complaints. 2. The objective section refers to information the physical therapist observes. Common examples include range of motion measurements, muscle strength, and functional abilities. 3. The assessment section allows the physical therapist express their opinion. Short and long-term goals are often expressed in this section as well as changes in the treatment program. 4. The plan section includes ideas for future physical therapy sessions. Frequency and expected duration of physical therapy services can also be incorporated into this section.

A physical therapist notices that a patient status post sternotomy has significantly altered their breathing pattern. The therapist is concerned that the alteration in breathing may lead to atelectasis. What is the MOST appropriate intervention to treat and prevent atelectasis? • 1. Active cycle of breathing • 2. Inspiratory muscle training • 3. Incentive spirometry • 4. Postural drainage

• Answer: 3 • Rationale: 1. The active cycle of breathing technique was developed under the name "forced expiratory technique" and is most commonly used to assist secretion clearance in patients with asthma. It includes three phases: breathing control, thoracic expansion exercises, and forced expiratory technique. This technique would not be used to prevent atelectasis 2. Inspiratory muscle training (IMT) uses a handheld breathing training device to provide resistance to increase the strength and endurance of the muscles of inspiration. IMT is not used to prevent atelectasis. 3. When performing a sustained maximal inspiration with an incentive spirometer, a maximal inspiratory effort is held at the point of maximum inspiration before exhalation. Prolonging and sustaining the maximum inspiration promotes recruitment of inspiratory muscle fibers and the drop in intrathoracic pressure enhances distribution of inhaled air. The technique promotes air passage past mucus obstructions in airways and maximizes alveolar expansion. Incentive spirometry is the most appropriate intervention to use to prevent atelectasis after sternotomy. 4. Postural drainage consists of positioning the patient so that gravity will help drain bronchial secretions from specific lung segments toward the gravity-dependent central airways where they can be removed by cough or mechanical aspiration

A physical therapist examines a patient with a complete C6 spinal cord injury. Which muscle would NOT be innervated based on the patient's level of injury? • 1. Biceps • 2. Deltoid • 3. Triceps • 4. Diaphragm

• Answer: 3 • Rationale: 1. The biceps muscle is innervated by the musculocutaneous nerve (C5-C6). 2. The deltoid muscle is innervated by the axillary nerve (C5-C6). 3. The triceps muscle is innervated by the radial nerve (C7-C8). 4. The diaphragm is innervated by the phrenic nerve (C3-C5).

A physician hypothesizes that a delayed union may be the result of disrupted blood supply. Which fracture would be MOST consistent with this clinical scenario? • 1. Calcaneus fracture • 2. Intertrochanteric hip fracture • 3. Scaphoid fracture • 4. Proximal humerus fracture

• Answer: 3 • Rationale: 1. The calcaneus is a quadrangular bone at the back of the tarsus. The bone articulates with the cuboid and the talus. Calcaneus fractures are commonly associated with high impact crush injuries such as a car crash or a fall from a ladder. Complications include pain, loss of motion, and arthritic changes, however, blood supply is not typically disrupted. 2. Intertrochanteric hip fractures are extracapsular and therefore do not affect blood supply. Conversely, femoral neck fractures are intracapsular and may lead to a disruption of the blood supply to the femoral head 3. The scaphoid is the largest carpal bone in the proximal row and is commonly injured when falling on an outstretched hand (FOOSH). Localized tenderness in the anatomic snuffbox is the most typical presentation associated with this type of injury. Scaphoid fractures can be particularly problematic since disruption in the scaphoid's blood supply can result in vascular necrosis. 4. Proximal humerus fractures are associated with high energy trauma in younger individuals and low energy falls in elderly individuals with osteoporotic bone. Over 80 percent of proximal humerus fractures are non-displaced and heal without surgery. Complications include shoulder stiffness, arthritis, and impaired function. Blood supply is not typically disrupted with this type of fracture.

A physical therapist palpates the bony structures of the wrist and hand. Which of the following structures would be identified in the proximal row of carpals? • 1. Capitate • 2. Hamate • 3. Triquetrum • 4. Trapezoid

• Answer: 3 • Rationale: 1. The capitate lies in the distal row of carpals and is located immediately proximal to the base of the third metacarpal. Wrist flexion can be used to facilitate palpation of the capitate. 2. The hamate is a wedge-shaped bone located in the distal row of carpals. The bone is distinguished by the hook-like process which projects from the volar surface and is attached to the flexor retinaculum. 3. The triquetrum is located on the medial side of the proximal row of carpals between the lunate and pisiform. 4. The trapezoid is the smallest carpal bone located in the distal row of carpals. The distal surface of the bone articulates with the proximal end of the second metacarpal bone and the proximal surface articulates with the scaphoid.

A physical therapist observes a patient complete hip abduction and adduction exercises in standing. Which axis of movement is utilized with these particular motions? • 1. Frontal • 2. Vertical • 3. Anterior-posterior • 4. Longitudinal

• Answer: 3 • Rationale: 1. The frontal (coronal) plane divides the body into anterior and posterior sections. Motions in the frontal plane occur around an anterior-posterior axis. Although the described motions, hip abduction and adduction, occur in the frontal (coronal) plane, the question specifically asks about the axis of movement. 2. The term vertical, or longitudinal, axis is used when the axis of motion passes through the length of a long bone. Motions in the transverse plane, such as medial and lateral rotation, occur around a vertical axis. The transverse plane divides the body into upper and lower sections. 3. Motions in the frontal (coronal) plane, such as abduction and adduction, occur around an anterior-posterior axis. The frontal plane divides the body into anterior and posterior sections. 4. The term longitudinal and vertical are synonyms and therefore refer to the same axis of movement

A physical therapist examines the integrity of the L4 spinal level on a patient. What deep tendon reflex should the therapist use to assess the L4 spinal level? • 1. Lateral hamstrings • 2. Medial hamstrings • 3. Patellar • 4. Achilles

• Answer: 3 • Rationale: 1. The lateral hamstrings reflex is innervated at the S1-S2 spinal level 2. The medial hamstrings reflex is innervated at the L5-S1 spinal level 3. The patellar reflex is innervated at the L3-L4 spinal level. 4. The Achilles reflex is innervated at the S1-S2 spinal level.

A patient presents with a small, round, partial-thickness wound on the lateral malleolus with distinct wound edges. Upon examination, the wound bed is a grey color without evidence of granulation tissue and appears to be dry. The wound is extremely tender to palpation and the patient reports calf pain during walking. These findings are MOST associated with which of the following types of ulcers? • 1. Venous stasis • 2. Neuropathic • 3. Arterial • 4. Decubitus

• Answer: 3 • Rationale: 1. Venous stasis ulcers result from venous insufficiency due to impaired functioning of the venous system, which results in inadequate venous return and eventual tissue damage and ulceration. Venous ulcers typically present as shallow and irregular in shape, have moderate to heavy exudate, and are usually located on the lower limb proximal to the medial malleolus. The involved lower limb is often edematous and trophic changes are common, especially a dark purple or rusty color associated with hemosiderin staining. 2. Neuropathic ulcers are a secondary complication usually associated with a combination of ischemia and neuropathy. Neuropathic ulcers are most often associated with diabetes mellitus, however, any form of peripheral neuropathy poses an increased risk of wound development. Neuropathic ulcers are most commonly located on the plantar aspect of the foot in areas susceptible to pressure or shear forces during weight bearing. Neuropathic ulcers are typically not painful due to a loss of protective sensation. 3. Arterial ulcers are the result of arterial insufficiency secondary to inadequate circulation of oxygenated blood (e.g., ischemia), often due to complicating factors such as atherosclerosis. Arterial ulcers are often small. round. With distinct wound margins and the wound bed typically lacks granulation tissue due to the lack of blood supply. Pain is often severe and arterial ulcers are typically located on the distal limb and foot, especially the toes, the dorsal surface of the foot or the lateral malleolus. 4. Decubitus ulcers are synonymous with pressure ulcers and are the result of sustained or prolonged pressure on tissue at levels greater than that of capillary pressure. Skin covering bony prominences is particularly susceptible to localized ischemia and tissue necrosis due to pressure. Although the wound in this scenario is over a bony prominence, the patient reports calf pain during walking, making it unlikely that this wound is a decubitus ulcer.

A physical therapist treats a patient who has a C6 spinal cord injury. Which of the following muscles would NOT be innervated based on the patient's level of injury? • 1. Biceps • 2. Deltoid • 3. Triceps • 4. Diaphram

• Answer: 3. • Rationale: 1. The biceps muscle is innervated by the musculocutaneous nerve (C5-C6). 2. The deltoid muscle is innervated by the axillary nerve (C5-C6). 3. None 4. A patient with a C6 spinal cord injury would not possess motor, sensory or reflex function below the C6 level. As a result, any muscle or structure innervated below this level would not be active.

A physical therapist monitors the blood pressure of a patient during increasing levels of physical exertion. Assuming a normal physiologic response, which of the following BEST describes the patient's blood pressure response to dynamic exercise? • 1. Systolic pressure increases, diastolic pressure increases • 2. Systolic pressure remains the same, diastolic pressure increases • 3. Systolic pressure and diastolic pressure remain the same • 4. Systolic pressure increases, diastolic pressure remains the same

• Answer: 4 • Rationale: 1. While an increase in systolic blood pressure is a normal response to dynamic exercise, an increase in diastolic blood pressure is an abnormal response. 2. No change in systolic blood pressure and an increase in diastolic blood pressure are both abnormal responses. An increase in diastolic blood pressure of more than 10 mm Hg may be indicative of exertional ischemia. 3. No change in either systolic or diastolic blood pressure is an abnormal response. 4. An increase in systolic blood pressure while diastolic blood pressure remains the same is a normal response to dynamic exercise.

A physical therapist works with a patient on proprioceptive and balance activities due to a history of recent falls. The patient informs the therapist that they have recently been diagnosed with optic neuritis. Which of the following medical conditions should the therapist MOST suspect might be the cause of the patient's clinical presentation? • Parkinson's disease • Guillan-Barre syndrome • Myasthenia gravis • Multiple sclerosis

• Answer: 4 1. The initial symptoms of multiple sclerosis (MS) can include visual problems, sensory changes, clumsiness, weakness, ataxia, balance dysfunction, and fatigue. Optic neuritis is often one of the first clinical signs of MS, since MS is characterized by demyelination of the myelin sheaths that surround the nerves within the brain and spinal cord. A history of recent falls, proprioceptive and balance impairments, and the presence of optic neuritis result in MS being the most likely medical condition. • Rationale: 1. The early signs of Parkinson's disease include resting tremors, usually in the hands (e.g., pill-rolling tremor) or feet, that increase with stress and disappear with movement or rest. Although early symptoms can also include balance disturbances, optic neuritis is not associated with Parkinson's disease. 2. The early signs of Guillain-Barre syndrome (GBS) include distal symmetrical motor weakness and mild distal sensory impairments. Complications most associated with GBS include muscle and respiratory paralysis, dysphagia, bladder weakness, and autonomic nervous system involvement including arrhythmia, tachycardia, postural hypotension, heart block, and absent reflexes. Optic neuritis is not associated with GBS 3. The primary feature of myasthenia gravis is muscle weakness within the skeletal muscles, with other neurologic findings being normal (e.g., reflexes, sensation). Because the ocular muscles are commonly affected, the patient can experience diplopia and ptosis (i.e., drooping eyelid), however, optic neuritis is not associated with myasthenia gravis. 4. The initial symptoms of multiple sclerosis (MS) can include visual problems, sensory changes, clumsiness, weakness, ataxia, balance dysfunction, and fatigue. Optic neuritis is often one of the first clinical signs of MS, since MS is characterized by demyelination of the myelin sheaths that surround the nerves within the brain and spinal cord. A history of recent falls, proprioceptive and balance impairments, and the presence of optic neuritis result in MS being the most likely medical condition.

A physical therapist applies a dressing to an area of skin on a patient's heel. The therapist decided to use the dressing as a prophylactic measure to reduce the risk of skin breakdown in an area that was determined to be particularly susceptible. Which of the following dressings would the therapist have MOST likely used? • 1. Calcium alginate • 2. Hydrocolloid • 3. Hydrogel • 4. transparent film

• Answer: 4 • Rationale: 1. A calcium alginate dressing is highly absorptive and typically utilized with wounds that produce moderate to heavy exudate. A wound producing minimal exudate is unlikely to saturate the alginate to the extent necessary for it to form a beneficial hydrophilic gel. 2. Hydrocolloid dressings consist of gel-forming polymers (e.g., carboxymethylcellulose, gelatin, pectin) backed by a strong film or foam adhesive. The dressings absorb exudate by swelling into a gel-like mass and vary in permeability, thickness, and transparency. A hydrocolloid dressing is not used for a superficial wound, however, is often used on partial and full-thickness wounds. 3. Hydrogel dressings are moisture-retentive primary dressings that provide a moist environment for wound healing. A hydrogel dressing is not typically used as a prophylactic measure and is instead used to prevent a wound from dehydrating and impeding the healing process. 4. Transparent film dressings consist of thin membranes coated with a layer of acrylic adhesive. Since the film is transparent it allows for frequent assessment of the wound and offers some level of protection. The films are oxygen permeable, however, are impermeable to microorganisms and moisture

A physical therapist is asked to assist in administering a graded exercise test to a patient referred to a phase Il cardiac rehabilitation program. Which of the following objective findings would MOST warrant the termination of the test? • 1. Diastolic blood pressure rising to 100 mm Hg • 2. Systolic blood pressure rising to 200 mm Hg • 3. Multifocal premature ventricular contractions • 4. Sustained ventricular tachycardia

• Answer: 4 • Rationale: 1. A diastolic blood pressure of 100 mm Hg may be indicative of a hypertensive response. However, diastolic blood pressure readings greater than 115 mm Hg are considered relative, not absolute, indications for terminating a graded exercise test. 2. A systolic blood pressure of 200 mm Hg may be a normal response to exercise, however, a measure greater than 250 mm Hg would be considered a relative indication for terminating a graded exercise test. 3. Multifocal premature ventricular contractions (PVCs) may be indicative of multiple areas within the ventricles that have become irritable due to hypoxia and as a result are producing ectopic beats. The presence of multifocal PVCs is a relative indication for terminating a graded exercise test. 4. Sustained ventricular tachycardia is an absolute indication for terminating a graded exercise test. Ventricular tachycardia lasting greater than 30 seconds is considered to be life-threatening.

A physical therapist observes a patient exercising on a treadmill while monitoring the patient's vital signs. The patient suddenly develops shortness of breath and chest pain. After resting, the patient develops a cough that produces blood-tinged sputum in addition to their other symptoms. The patient is MOST likely experiencing which condition? • 1. Unstable angina • 2. Myocardial infarction • 3. Pneumothorax • 4. Pulmonary embolism

• Answer: 4 • Rationale: 1. Angina is a condition described as a feeling of pressure and/or pain that commonly occurs in the chest, though can extend to the epigastric region or jaw. Angina occurs when the oxygen demand of the myocardium exceeds its blood supply. Unstable angina is characterized by angina symptoms that occur at rest. Angina would not be associated with shortness of breath or sputum-producing cough 2. Myocardial infarction is a condition characterized by chest pressure and/or pain that can radiate to the back, arm, or jaw. Other symptoms may include shortness of breath, nausea, vomiting, diaphoresis, or syncope. Myocardial infarction is caused by a blockage of blood flow to the myocardium and is considered a medical emergency. A cough that produces blood-tinged sputum is not a symptom commonly associated with myocardial infarction. 3. Pneumothorax is a condition in which air is able to enter the pleural space, usually secondary to a traumatic injury involving a penetration wound. Symptoms include chest pain and shortness of breath, as well as cyanosis and anxious behavior. A cough that produces blood-tinged sputum is not a symptom commonly associated with pneumothorax 4. Pulmonary embolism is a condition that occurs when a thrombus loosens, travels through the circulatory system, and eventually becomes lodged in the lungs. Symptoms include chest pain that may worsen with coughing or deep breathing, shortness of breath, tachycardia, a cough that produces blood-tinged sputum, diaphoresis, and syncope. Pulmonary embolism is considered a medical emergency that requires immediate contact with emergency medical services.

A physical therapist is treating a patient with right hemiplegia in standing balance activities. The therapist works on reaching activities on level surfaces, on uneven surfaces, and by picking items up off the ground. Which type of practice is MOST consistent with the described interventions? • 1. Blocked • 2. Distributed • 3. Constant • 4. Variable

• Answer: 4 • Rationale: 1. Blocked practice refers to the consistent practice of a single task. If the therapist was using a blocked practice method, the patient would perform the same single task repeatedly to improve standing balance. 2. Distributed practice refers to a method of practice where a patient's rest time between trials is equal to or greater than the practice time itself. 3. Constant practice refers to the practice of a given task under a uniform condition. If the therapist was using a constant practice method, the patient would perform standing balance activities in the same environment and with the same equipment for each trial. 4. Variable practice refers to the practice of a given task under differing conditions. The described scenario best demonstrates variable practice since the patient is working on standing balance activities on level surfaces, on uneven surfaces, and when attempting to obtain items from the ground.

A physical therapist prepares to measure the blood pressure of a patient with suspected hypertension. Which action by the patient would be unnecessary when conducting the measurement? • Avoid caffeine for 30 minutes before the measurement • Sit with the arm resting on a table during the measurement • Sit quietly for five minutes before the measurement • Avoid eating for one hour before the measurement

• Answer: 4 • Rationale: 1. Caffeine is a central nervous system stimulant that can cause short-term increases in blood pressure. Avoiding caffeine for 30 minutes before the measurement is obtained would provide a more valid measurement of the patient's blood pressure. 2. When performing a blood pressure measurement, the patient should have their arm resting at roughly chest height. Having the arm in a position higher or lower than chest height can result in an erroneous blood pressure measurement. It is also important that the patient does not actively hold their arm in this position as muscle contraction can cause an increase in blood pressure. 3. Activity has the potential to increase blood pressure. As a result, having the patient sit quietly for a minimum of five minutes before the blood pressure measurement is obtained allows the patient to return to their normal resting state. 4. Avoiding food for one hour before the blood pressure measurement is obtained is unnecessary since the presence of food in the body has minimal effects on blood pressure compared to the other presented options.

A physical therapist treats a patient referred to an outpatient rehabilitation facility. During the session, the therapist identifies weakness, hypertonia, and hyperreflexia. What is the MOST likely diagnosis? • 1. Carpal tunnel syndrome • 2. Poliomyelitis • 3. Bell's palsy • 4. Multiple sclerosis

• Answer: 4 • Rationale: 1. Carpal tunnel syndrome is a lower motor neuron pathology comprised of peripheral nerve entrapment that occurs as a result of compression of the median nerve where it passes through the carpal tunnel. This results in sensory and motor disturbances in the median nerve distribution of the hand 2. Poliomyelitis is a lower motor neuron pathology comprised of a viral infection resulting in neuropathy that includes focal and asymmetrical motor impairments. Post-polio syndrome is a lower motor neuron pathology that affects the anterior horn cells of those previously affected with polio. Post-polio syndrome occurs when the compensated reinnervation fails and results in ongoing muscle denervation. 3. Bell's palsy is a lower motor neuron pathology comprised of temporary unilateral facial paralysis secondary to trauma with demyelination and/or axonal degeneration of the facial nerve. Inflammation and subsequent pressure injure the nerve with varying degrees of damage. The inflammation within the auditory canal produces subsequent demyelination of the nerve and if ischemia occurs, there is axonal degeneration of the nerve. 4. Multiple sclerosis is an upper motor neuron pathology that produces patches of demyelination of the myelin sheaths that surround nerves within the brain and spinal cord. This decreases the efficiency of nerve impulse transmission and symptoms will vary based on the location and the extent of demyelination. There is subsequent plaque development and eventual failure of impulse transmission.

A physical therapist listens to the lung sounds of a patient with chronic bronchitis. The patient was admitted to the hospital two days ago after complaining of shortness of breath and difficulty breathing. While performing auscultation, the therapist identifies distinct lung sounds with a high constant pitch during exhalation. What is this type of sound MOST consistent with? • 1. Crackles • 2. Rales • 3. Rhonchi • 4. Wheezes

• Answer: 4 • Rationale: 1. Crackles are discontinuous, adventitious breath sounds heard during auscultation of the lungs due to fluid accumulation in the distal airways or when collapsed alveoli reopen during inspiration. 2. Rales are synonymous with crackles. 3. Rhonchi are lower-pitched, continuous, adventitious breath sounds occurring during inspiration or expiration and are caused by the turbulence of air passing through secretions in large and mid-sized bronchi. 4. Wheezes are continuous, high-pitched, adventitious breath sounds that are most frequently heard on exhalation and are associated with airway obstruction or bronchospasm. There can also be inspiratory wheezing caused by movement of air through secretions. Wheezes are commonly associated with asthma and bronchitis

A patient rehabilitating from a lower extremity injury has been non-weight bearing for three weeks. A recent physician entry in the medical record indicates the patient is cleared for weight-bearing up to 25 pounds. What is the MOST appropriate device to use when instructing the patient on the new weight-bearing status? • 1. An inclinometer • 2. A tape measure • 3. An anthropometer • 4. A scale

• Answer: 4 • Rationale: 1. Inclinometers, also termed gravity-dependent goniometers, use gravity's effect on pointers and fluid levels to measure joint position and motion. 2. Tape measures are designed to quantify distance. A common use in physical therapy would be to assess leg length or conduct girth measurements. 3. An anthropometer is a device used to gather data on the measurements and proportions of the human body. Common uses include measuring long bone length, skin folds or use in motion analysis studies. 4. A scale can be a valuable tool for a patient to use to better understand what a selected amount of weight-bearing feels like since it offers immediate feedback in the form of pounds.

A physical therapist attempts to determine a patient's general willingness to use an affected body part. What objective information would be the MOST useful? • 1. Passive movement • 2. Sensory testing • 3. Bony palpation • 4. Active movement

• Answer: 4 • Rationale: 1. Passive movement refers to the arc of motion attained by a therapist without assistance from the patient. Passive range of motion is generally slightly greater than active range of motion. The activity provides the therapist with information on the integrity of the articular surfaces, extensibility of the joint capsule, ligaments, muscles, fascia, and skin. 2. Sensory testing is an umbrella term that includes the examination of superficial sensations, deep sensations, and combined cortical sensations. The vast majority of sensory tests would require the patient to verbalize and would not require the active use an affected body part. 3. Bony palpation is a passive technique that would not require the patient to actively participate. 4. Active movement requires the patient to perform unassisted voluntary range of motion. The activity provides the therapist with information on the patient's willingness to use the affected body part, available range of motion, strength, and coordination.

A physical therapist discusses common cognitive and behavioral changes associated with stroke with family members of a patient with right hemisphere damage and resultant left hemiplegia. Which term does NOT accurately describe the typical patient presentation? • 1. Poor judgment • 2. Impulsive • 3. Quick • 4. Overly cautious

• Answer: 4 • Rationale: 1. Patients with right hemisphere damage often have difficulty in grasping the whole idea of a task or activity and therefore may have difficulty with reasoning and judgment. 2. Patients with right hemisphere damage are often impulsive and tend to overestimate their abilities and underestimate their limitations. This combination makes the patient a greater safety risk compared to a patient with left hemisphere damage. 3. Patients with right hemisphere damage often are quick when performing mobility or tasks. Feedback must be given to encourage the patient to slow down and focus on each of the sequential steps of the activity 4. Patients with left hemisphere damage often are described as cautious, anxious, and disorganized. They often have difficulties communicating and in processing information in a sequential manner.

A physical therapist is treating a patient with an acute ankle sprain. The therapist plans to utilize elastic bandaging to reduce the patient's ankle edema. Which of the following bandages would be the MOST appropriate to use with this patient? • 1. Short-stretch bandaging to utilize a high working pressure • 2. Short-stretch bandaging to utilize a high resting pressure • 3. Long-stretch bandaging to utilize a high working pressure • 4. Long-stretch bandaging to utilize a high resting pressure

• Answer: 4 • Rationale: 1. Short-stretch bandages have a high working pressure and a low resting pressure. Short-stretch bandages are often utilized to manage edema due to venous disease and lymphedema and to assist the muscle pumping action of the calf muscles while the patient is active. 2. Short-stretch bandages have a high working pressure, not a high resting pressure. 3. While long-stretch bandages are desirable for controlling edema after an ankle sprain, they have a high resting pressure, not a high working pressure. 4. Long-stretch bandages are often used following an ankle sprain to assist with controlling edema using their high resting pressure.

A patient who has cerebral palsy exhibits slow, involuntary, continuous writhing movements of the upper and lower extremities. Which of the following motor impairments is MOST representative of this type of motor disturbance? • 1. Spasticity • 2. Ataxia • 3. Hypotonia • 4. Athetosis

• Answer: 4 • Rationale: 1. Spasticity refers to an increased resistance to passive stretch. Spasticity is commonly observed with patients diagnosed with cerebral palsy due to upper motor neuron damage. 2. Ataxia is a generalized term used to describe motor impairments of cerebellar origin. It is characterized by the inability to perform coordinated movement and may affect gait, posture, and patterns of movements. 3. Hypotonia refers to a decreased or absent tone where resistance to passive movement is decreased, stretch reflexes are diminished, and limbs are easily moved. Hypotonicity in children is often associated with motor delays. 4. Athetosis refers to involuntary movements characterized as slow, irregular, and twisting. Peripheral movements occur without central stability. This type of motor disturbance makes it extremely difficult to maintain a static body position.

A patient who has ankylosing spondylitis is referred to physical therapy for instruction in a home exercise program. Strengthening of which of the following muscles would be the MOST beneficial for the patient? • 1. Rectus abdominis • 2. Internal and external obliques • 3. Quadratus lumborum • 4. Erector spinae

• Answer: 4 • Rationale: 1. Strengthening of the rectus abdominis will produce a flexion moment in the trunk. This will further accentuate the thoracic kyphosis and decrease the lumbar lordosis 2. Strengthening of the internal and external obliques when contracting bilaterally will produce a flexion moment of the trunk and when contracting unilaterally, will produce a rotary moment of the trunk. General core strengthening is desirable, however, the emphasis would be on strengthening the back extensors. 3. Strengthening of the quadratus lumborum will produce a lateral bending of the trunk when performed in a closed-chain activity and a hip hiking movement when performed in an open-chain activity. These motions would not be the emphasis of a strengthening program for a patient with ankylosing spondylitis. The quadratus lumborum also assists with extension, however, the muscle is just one of many muscles that serve this function. 4. Extension exercises are often an important component of a comprehensive plan of care to assist patients with ankylosing spondylitis to maintain the normal curves of the spine while at the same time limiting the forward bending nature of the disease process


Kaugnay na mga set ng pag-aaral

Chapter 12 - Individual Policy Provisions

View Set

6. Mid-Term Pre-Assessment Review Guide

View Set